617
Árki Tamás Konfárné Nagy Klára Kovács István Trembeczki Csaba Urbán János Mozaik Kiadó – Szeged, 2010 11 FELADATGYÛJTEMÉNY sokszínû MEGOLDÁSOK

11. Példatár megoldások

  • Upload
    vanbao

  • View
    768

  • Download
    29

Embed Size (px)

Citation preview

  • rki TamsKonfrn Nagy KlraKovcs IstvnTrembeczki CsabaUrbn Jnos

    Mozaik Kiad Szeged, 2010

    11FELADATGYJTEMNY

    s o k s z n

    MEGOLDSOK

    matematika_11_fgy_mo.qxd 2010.11.16. 9:07 Page 1

  • TARTALOMJEGYZK

    Megoldsok 11. vfolyam

    11.1. Kombinatorika, grfok (3001-3160)Fibonacci-szmok .................................................................................................................. 4Permutcik, varicik ......................................................................................................... 5Ismtls nlkli kombincik, Pascal-hromszg ...................................................... 7Binomilis egytthatk, ismtlses kombinci ......................................................... 13Vegyes sszeszmllsi feladatok (kiegszt anyag) .............................................. 15GRFOK pontok, lek, fokszm .................................................................................... 19GRFOK t, vonal, sta, kr, Euler-vonal (kiegszt anyag) ............................. 23Fagrfok (kiegszt anyag) ................................................................................................ 25A kombinatorika gyakorlati alkalmazsai ....................................................................... 29Vegyes feladatok ..................................................................................................................... 30

    11.2. Hatvny, gyk, logaritmus (3161-3241)Hatvnyozs s gykvons (emlkeztet) .................................................................... 33Hatvnyfggvnyek s gykfggvnyek ........................................................................ 34Trtkitevj hatvny .............................................................................................................. 38Irracionlis kitevj hatvny, exponencilis fggvny .............................................. 39Exponencilis egyenletek, egyenletrendszerek,

    egyenltlensgek ........................................................................................................... 45A logaritmus fogalma ............................................................................................................ 50A logaritmusfggvny ........................................................................................................... 53A logaritmus azonossgai ................................................................................................... 58Logaritmikus egyenletek, egyenletrendszerek,

    egyenltlensgek ........................................................................................................... 60Vegyes feladatok ..................................................................................................................... 66

    11.3. A trigonometria alkalmazsai (3242-3459)Vektormveletek rendszerezse, alkalmazsok (emlkeztet) .............................. 72A skalris szorzat ................................................................................................................... 74Skalris szorzat a koordinta-rendszerben ................................................................... 78A szinuszttel ........................................................................................................................... 82A koszinuszttel ...................................................................................................................... 87Trigonometrikus sszefggsek alkalmazsai ............................................................. 91

    TARTALOMJEGYZK

    2

    matematika_11_fgy_mo.qxd 2010.11.16. 9:07 Page 2

  • sszegzsi kpletek .............................................................................................................. 98Az sszegzsi kpletek alkalmazsai .............................................................................. 104Trigonometrikus egyenletek, egyenletrendszerek ....................................................... 112Trigonometrikus egyenltlensgek ................................................................................... 130Vegyes feladatok ..................................................................................................................... 135

    11.4. Fggvnyek (3460-3554)Az exponencilis s logaritmusfggvny ....................................................................... 148Egyenletek s fggvnyek ................................................................................................... 156Trigonometrikus fggvnyek .............................................................................................. 167Trigonometrikus egyenletek, egyenltlensgek

    (kiegszt anyag) ......................................................................................................... 171Vegyes feladatok ..................................................................................................................... 179Inverz fggvnyek (kiegszt anyag) ............................................................................. 193

    11.5. Koordinta-geometria (3555-3776)Vektorok a koordinta-rendszerben. Mveletek koordintikkal

    adott vektorokkal (emlkeztet) ................................................................................ 198Kt pont tvolsga. Kt vektor hajlsszge.

    Terletszmtsi alkalmazsok .................................................................................. 201Szakasz osztpontjnak koordinti. A hromszg

    slypontjnak koordinti ........................................................................................... 207Az egyenest meghatroz adatok a koordinta-rendszerben ................................. 216Az egyenes egyenletei .......................................................................................................... 222Kt egyenes metszspontja, tvolsga, hajlsszge ................................................ 233A kr egyenlete ........................................................................................................................ 241A kr s az egyenes klcsns helyzete;

    kt kr kzs pontjai ..................................................................................................... 252A parabola ................................................................................................................................. 264Vegyes feladatok ..................................................................................................................... 276

    11.6. Valsznsg-szmts, statisztika (3777-3892)Klasszikus valsznsgi modell ....................................................................................... 288Visszatevses mintavtel ..................................................................................................... 295Mintavtel visszatevs nlkl (kiegszt anyag) ........................................................ 301Valsznsgi jtkok grfokon (kiegszt anyag) .................................................... 305Valsg s statisztika ............................................................................................................ 309Vegyes feladatok ..................................................................................................................... 309

    3

    TARTALOMJEGYZK

    matematika_11_fgy_mo.qxd 2010.11.16. 9:07 Page 3

  • MEGOLDSOK 11 . VFOLYAM

    4

    11.1. KOMBINATORIKA, GRFOK

    Fibonacci-szmok megoldsok

    w x3001 f15 = 610, f20 = 6765.

    w x3002 a) 1, 1, 0, 1, 1, 2, 3, 5, 8, 13;b) 1, 1, 0, 1, 1, 2, 3, 5, 8, 13.

    w x3003 a) 1, 1, 2, 4, 7, 13, 24, 44, 81, 149, 274, 504, 927, 1705, 3136.b) Igen, a 13.

    w x3004 A msodik tagot az els s az els eltt ll sszegeknt kapjuk: x + 1 = 1; innen x = 0. Az elttelev tag: y + 0 = 1; innen y = 1. Majd z + 1 = 0; z = 1. Aztn u + (1) = 1; u = 2. Hasonlankapjuk a tbbi szmot:

    34, 21, 13, 8, 5, 3, 2, 1, 1, 0, 1, 1, 2, 3, 5, rdekes mdon a Fibonacci-szmokat ltjuk, csak vltakoz eljellel.

    w x3005 rdemes elszr ksrletezni nhny rtkkel. Hamar megtalljuk a csupa 0 sorozatot, amiismtldst mutat. Azonban ms sorozat nem, gy megfogalmazhatjuk a sejtsnket: ltalban nemlehet a szmok kztt ismtlds. St, egy id utn vagy szigoran nvekednek, vagy szigorancskkennek a szmok a mr emltett csupa 0 kivtelvel. Hogyan bizonytsuk be?Gyjtsnk ssze egyszer megfigyelseket.A) Ha kt pozitv szm van egyms utn valahol a sorozatban, akkor az utnuk levk is pozitvak,

    st a szmok szigoran nvekednek az sszeads miatt.B) Ha van kt szomszdos negatv szm a sorozatban, akkor az utnuk levk is negatvak, st

    a szmok szigoran cskkennek.C) Ha tallunk nullt, s eltte pozitv vagy negatv rtket, akkor az A) vagy B) esethez jutunk:

    a nulla megduplzza az eltte ll szmot.Jnak tnik a gondolatmenet, folytassuk. Mi a helyzet, ha egy pozitv s egy negatv rtk ll egy-ms mellett?D) Ha a kt szm abszolt rtke egyenl, akkor megjelenik a nulla, C) esethez jutunk.E) Ha a pozitv szm abszolt rtke nagyobb, akkor pozitv rtket kapunk. Ezen bell E1),

    ha a pozitv rtk volt ksbb, az A) esethez jutottunk.F) Ha a negatv rtk abszolt rtke nagyobb, akkor negatv szmot kapunk. Ezen bell F1),

    ha a negatv rtk volt ksbb, a B) esethez jutottunk.Az E) s F) esetben van egy-egy tovbbi eset is: E2), ha a negatv rtk van ksbb; illetve F2),ha a pozitv rtk szerepel ksbb. Mindkt helyzetben visszajutunk D), E) vagy F) esethez.Radsul a ltrejv szm abszolt rtke megegyezik az sszeg abszolt rtkvel, ami szigorankisebb lesz, mint az elz kett kzl a nagyobb abszolt rtk. Ennek a cskkensnek pedigelbb-utbb vge szakad: A), B) vagy C) esetekhez kanyarodunk vissza.Nhny plda az E2) s F2) esetekre:a) 10, 4, 6, 2, b) 10, 8, 2, 6, 4, c) 10, 5, 5, 0, Mivel minden esetet ttekintettnk, ezzel a bizonytst befejeztk: a 0, 0, 0, szmok kivtelvelnincs ms olyan Fibonacci-szer sorozat, amiben lenne ismtlds (st, elbb-utbb mindig szigo-ran nv vagy szigoran cskken szmokat kapunk).

    matematika_11_fgy_mo.qxd 2010.11.16. 9:07 Page 4

  • Permutcik, varicik megoldsok

    w x3006 86!

    w x3007 27!

    w x3008

    w x3009 a) b) c) 0! = 1.

    w x3010

    w x3011 a)

    b)

    w x3012 107.

    w x3013 28 = 256.

    w x3014 Ebben a feladatban s a tovbbiakban is nemnegatv (st elssorban pozitv) egsz megoldsokatkeresnk. A legegyszerbb, ha prblkozunk.A msik lehetsg, hogy kicsit gondolkodunk a prblkozs eltt: n! = 39 916 800. A szm vgnll kt 0 arra utal, hogy kt ts prmtnyeznek lennie kell a szorzatban. Az egyik maga az 5.A msikat a 10-ben talljuk, teht legalbb 10!-ig el kell menni. Ez mg kevs, a megolds n = 11.

    w x3015 Az els hrom szmjegyet 63, a msodik hrmat -flekppen kaphatjuk meg. Az ered-mny a kett szorzata: 216 720 = 155 520.

    w x3016 a) Egyszer permutcija 8 klnbz elemnek: 8! = 40 320.b) Vlasszunk ki egy ft, tekintsk t kiindulpontnak. A megadott krljrs szerint ltessk

    sorba a maradk ht ft. Ez gy trtnik, mintha egyszeren permutlnnk ket: 7! = 5 040.c) Most nem rdekes a krljrsi irny, ezrt ugyanazt kapjuk, ha az asztal egy tmrjre tk-

    rzzk a rsztvevket. gy az elz pontban msik ltetst kaptunk volna, most azonban

    ez a kett ugyanaz. A megolds:

    w x3017 Szmoljuk meg a nvnyeket, sszesen 6-flt tallunk a pulton. Mindegyikbl minden helyentehet egyet a kosarba, azaz a 6n = 1679 616 egyenletet kell megoldanunk. A megoldst meg-kaphatjuk prblkozssal vagy az 1679 616 prmtnyezs felbontsval. A megolds: n = 8. Megjegyzs: Pr lecke mlva mindkt oldal 6-os alap logaritmust vve is meg tudjuk oldaniaz egyenletet.

    w x3018 A msodik dobozbl az ismtlds miatt a cdulkat -flekpp hzhatjuk ki (permutci,

    hiszen mindet kivesszk). Az els dobozbl ugyancsak az ismtlds miatt 104 lehetsgnk vankivenni a cdulkat (varici, a kivlaszts miatt). Eredmnynk a kett szorzata: 12 600 000.

    9!

    2! 3! 4!

    7!=

    22520.

    10!

    ( )!10 3

    10!= 10 9 8 7 =

    ( )!.

    10 45040

    10!= 10 9 8 7 6 5 =

    ( )!;

    10 6151200

    18!= 18 17 16 15 14 13 = 13366080.

    ( )!18 6

    ( )!;

    3 2 010

    + +

    3! 2! 0!=

    ( )!

    ;3 2

    10+

    3! 2!=

    21

    10162954792

    !

    !.

    6! 5!=

    KOMBINATORIKA , GRFOK

    5

    matematika_11_fgy_mo.qxd 2010.11.16. 9:07 Page 5

  • MEGOLDSOK 11 . VFOLYAM

    6

    w x3019 Egy n alap szmrendszerben 0-tl (n 1)-ig n klnbz szmjegy van, ezekbl kell 7 helyrerni egyet-egyet. (A kevesebb jegybl ll szmokat gy kapjuk, hogy a szm elejre megfelelszm 0-t runk.) Vagyis az n7 = 2 097 152 egyenletet kell megoldanunk, amit prblkozssalvagy hetedik gykvonssal tesznk meg. Az eredmny n = 8, a szmrendszer teht a nyolcas.

    w x3020 a) Brmikor brmelyik cscsra ugorhat, minden esetben ngy lehetsge van: 410.b) Elsnek brhova, utna viszont mr csak 3-3 helyre ugorhat: 4 39.c) Elsnek brhova ugorhat. A kvetkez ugrsa a mostani helyrl elviszi, az utna levkben

    pedig mindig kizrunk kett cscsot: 4 3 28.

    w x3021 Legyen a msor nzszma n. Ekkor az els jtkost n, a msodikat (n 1)-flekppen vlaszt-hatjk ki a nzk kzl, vagyis

    Ezt az egyenletet ngyflekpp: prblkozssal; a zrjelet felbontva msodfok egyenletknt;az 1980-at prmtnyezkre bontva vagy gykvonsbl kerektve is megoldhatjuk. Az eredmny n = 45.

    w x3022 a) A hrom emltett miniszterelnkt vegyk egy csomagnak. Ekkor 8 ft kell leltetnnk, ezt8!-flekpp tehetjk meg. Az angolnak s a francinak kzre kell fognia a nmetet, ket ezrtmg ktfelkppen ltethetjk minden sorrenden bell. Az eredmny: 2 8!

    b) Ismt csomagoljuk ssze a hrmast. 8 ft kr alak asztal mell 7!-flekpp ltethetnk le,ha figyelembe vesszk a krljrsi irnyt is. Vegyk figyelembe a hrmasban a szlsk sor-

    rendjt is: 2 7! Vgl tekintsnk el a krljrsi irnytl:

    w x3023 Egyik lehetsgnk a prblkozs. Msrszt viszont ha szmolunk, ismtlses permutcit kellszmolnunk. Jelljk c-vel (2 c < 12) a keresett sznpadi mvek szmt:

    Tntessk el a trtet, a kvetkez egyenletet kapjuk:(9 + c)! = 79 833 600 c!

    Leosztva 11!-sal (figyelembe vve c lehetsges rtkeit):

    A c rtktl fggen tbb esetnk van. Ha 9 + c = 12, akkor c = 3 s gy 12 = 2 3! Ez jmegolds.Ha 3 < c < 12, akkor a bal oldalon szerepel 13, ezrt a jobb oldalon is el kell menni legalbb13!-ig. Azonban ebben (jobb oldalon) szerepel 11 is, ami viszont a msik (bal) oldalon nem fogelfordulni, ellentmondsra jutottunk. Ezen esetekben nincs megolds, Lacinak 3 sznhzi eladsttartalmaz lemeze van.

    w x3024 Az t gyermek megszlethet 5 lny; 4 lny s 1 fi; 3 lny s 2 fi; 2 lny s 3 fi; 1 lny s 4 fi;5 fi variciban. Az els s az utols esetben egyszer dolgunk van, mindktszer 5! sorrendben adhattak neveta megszletett gyerekeknek. Vegyk bonyolultabb pldnak a harmadik esetet. Ekkor a gyerekek kztt a lnyok-fik sorrendje

    ismtlses permutci: azon bell a lnyoknak a fiknak -flekppen

    adhatnak nevet (ismtls nlkli varici).

    5!

    !( )5 2

    5!

    !( ),

    5 3

    5!

    3! 2!,

    12 13 9 26 13 9

    ( ) !( ) !

    + =+ =

    c cc c

    ( )

    !.

    4 527720

    + + !

    4! 5!=

    c

    c

    2 72

    7 !

    = !

    n

    nn n

    !

    != =

    ( )( ) .

    21 1980

    matematika_11_fgy_mo.qxd 2010.11.16. 9:07 Page 6

  • KOMBINATORIKA , GRFOK

    7

    Ez alapjn minden esetet szmba tudunk venni (a szimmetria miatt elg az egyes eseteket kettvelszorozni), a vgeredmny:

    w x3025 Elszr is gondoljuk t a feltteleket. Az ismtlses esetben k-ra s n-re semmifle megszortsnincs azon kvl, hogy nemnegatv egszek. Az ismtls nlkli esetben azonban 0 k n. Ez szi-gorbb felttel az elznl, gy tartsuk magunkat az utbbihoz. A krds: mely, a fenti feltteleknekmegfelel n, k-ra igaz:

    Felttelezhetjk, hogy n 0, gy nem kell a 00 hatvnnyal foglalkoznunk. Vizsgljuk meg k nhnyrtkt! Ha k = 0, akkor

    vagyis ekkor brmely n-re teljesl az egyenlsg. Ha k = 1, akkor ugyanez a helyzet:

    Ha k = 2, akkor n2 = n (n 1) egyenlethez jutunk. Ennek egyetlen megoldsa n = 0, ami nemfelel meg a k n felttelnek. Ha k rtkt tovbb nveljk, akkor egyre magasabb fok egyen-letekhez jutunk, melyeket nem tudunk megvizsglni. A megolds azonban sokkal egyszerbb.Azt kell szrevennnk, hogy a kiindul egyenlsg jobb s bal oldaln is k tnyezbl ll szor-zatokat tallunk, azonban a bal oldalon csupa n tnyezvel, a jobbon viszont n-tl cskkentnyezkkel. Vagyis ha a szorzatoknak tbb tnyezje is van, akkor nem lehetnek egyenlk.A megolds: k = 0 s k = 1 rtkre brmely n-re megegyezik n elem k-tag ismtlses s ismt-ls nlkli variciinak szma, ms rtkekre viszont soha.

    Ismtls nlkli kombincik, Pascal-hromszg megoldsok

    w x3026 a) 6; b) 105; c) 462; d) 165; e) 55; f ) 7381;g) 145; h) 1578; i) 1.

    w x3027 a) 1; b) n; c) n2 n; d) e) n; f ) 1.

    w x3028 a) b) c)

    w x3029 a) Nincs.b) Egy helyen: 2; kett helyen: 3 s 4; hrom helyen: 6; ngy helyen: 10. (Tbb helyen nem

    lehetnek, mert utoljra a szls egyes mellett fordulhatnak el.)1

    2

    6

    20

    70

    252

    1

    3

    10

    35

    126

    1

    4

    15

    56

    210

    1

    5

    21

    84

    1

    6

    28

    120

    1

    7

    36

    1

    8

    45

    1

    9

    1

    10

    1

    1

    1

    3

    10

    35

    126

    1

    4

    15

    56

    210

    1

    5

    21

    84

    1

    6

    28

    120

    1

    7

    36

    1

    8

    45

    1

    9

    1

    10

    1

    1

    11255

    .

    212101

    ;

    13544

    ;

    n n2

    2

    ;

    n nn

    nn= =

    !

    !=1

    1( ).

    1 10= =!

    !=n

    n

    n,

    nn

    n kk =

    !

    !( ).

    2 5 21 5 4 5 1

    22 5 3 5 2

    !! ( ) ( ) ! ( ) (

    +5!

    4!

    5!

    !

    5!

    !+

    5!

    3!

    5!

    !

    5!

    )).

    != 30240

    matematika_11_fgy_mo.qxd 2010.11.16. 9:07 Page 7

  • MEGOLDSOK 11 . VFOLYAM

    8

    w x3030 a) b) c) d)

    e) 6 elembl nem lehet 7-et kivlasztani.

    w x3031 a) 6; b) 5.

    w x3032

    w x3033 a) b)

    w x3034

    w x3035

    w x3036

    w x3037 A Pascal-hromszg ellltsbl addan szimmetrikus, s az rtkek a sorok elejn nvekedneka szimmetriatengelyig. gy ha pros szmrl van sz, a felt kell alrnunk. Pratlanokra kt megoldstis kapunk: egyik a szm felnek egszrsze, msik az ennl eggyel nagyobb egsz. A konkrt pldban:

    a) k = 12, b) k = 6 vagy k = 7,

    w x3038 a) A trpk kzl -flekpp vlogathat ebdre ngyet Hkuszpk.

    b) A maradk nyolc fbl vlasztanak elszr kettt, majd a maradk hatbl ismt kettt. Mivel

    ezeket egymstl fggetlenl meg tudjk tenni, ssze kell ket szoroznunk:

    w x3039 A lehetsgek szma:

    w x3040 a) A ngykerek aut mind a ngy kerekre ngy csavar kell, gy a vlasz:

    b) Ha kln-kln vesz ki csavarokat a dobozbl Sanyi, akkor a kvetkez adagot mindig nggyel

    kevesebb kzl vlaszthatja, -flekpp.

    c) Az a) eset kifejtve egyszer: Most b) esetet is fejtsk ki:

    Az egyszerstsek utn maradt alakokbl mivel 16! > (4!)4 mr lthat, hogy a b) esetbentbb lehetsgnk van a csavarok kivlasztsra. Vgl osszuk el a b)-ben kapott eredmnyt az a)-ban kapottal. gy -hoz jutunk,

    ami megfelel 16 elem 4, 4, 4, 4-tag ismtlses permutciinak. Azaz ha az a) esetben figye-lembe vesszk ezt a sorrendet is (vagy a b) esetben eltekintnk tle), akkor a kt eset egyenl lesz.

    16

    4 4 4 4

    !

    !! ! !

    604

    564

    524

    484

    604 56

    564 52

    524

    =! ! !

    !!

    !!

    !448

    484 44

    604 444!

    !!

    !( !

    . !

    =!)

    6016

    6016 44

    =!

    !!.

    604

    564

    524

    484

    6016

    .

    93

    42

    504 = .

    82

    62

    420 = .

    124

    495 =

    136

    137

    1716

    = = .

    2412

    2704156 = ;

    604

    487635 = .

    73

    35 = .

    93

    84 = .

    905

    456

    18 89 11 29 8615 44 43 7 41

    3 89 295 7 41

    5

    = = ,, .4456

    8145060 = ;

    74

    35 = .

    65

    6 = ;

    64

    15 = ;

    63

    20 = ;

    62

    15 = ;

    matematika_11_fgy_mo.qxd 2010.11.16. 9:07 Page 8

  • KOMBINATORIKA , GRFOK

    9

    w x3041 a) Ha a piros kirly a leosztott lapok kztt van, akkor a maradk 31-bl kell mg mell tenni

    hrmat:

    b) Ez az eset hrom egymst kizr alesetbl ll ssze: piros, de nem kirly; piros kirly; nempiros, de kirly. A hrom esetben sszesen 11 lap van (7 + 1 + 3), melyekbl bele kell kerlnieegynek a leosztott ngy lap kz. A tbbi hrom leosztott lap a maradk 21 lapbl kerlhet ki.

    A megolds:

    c) A legalbb sz gyans lehet, inkbb szmoljuk ki a krdezett eset ellenttt (komplementert).Ekkor arra kell vlaszolnunk: hnyflekpp fordulhat el, hogy a leosztott ngy lap kztt nincssem piros, sem kirly lap? Azokbl a lapokbl, melyek nem pirosak s kirly sincs rajtuk, 21 van.

    A megoldst gy kapjuk, ha kivonjuk az sszes esetbl az ellentettet:

    w x3042 Az autszerels feladathoz hasonl szorzatot kell felrnunk: Az ottanihoz

    hasonlan elvgezve az egyszerstseket:

    w x3043 I. megolds. A legalbb egy sz ellentte, ha nincs sz a leosztsban. Mivel sszesen ngy sz van

    a pakliban, gy az ellenttes eseteket az sszes esetbl kivonva a megolds:

    II. megolds. A legalbb egy jelent egy, kett, hrom vagy ngy szt. Ez ngy eset, lssuk rsz-letesebben az egyiket, mondjuk a hrom szt. Ekkor a csomagban lev ngybl hrom bekerl

    a leosztsba, a maradk egyet pedig a nem szok kzl tltjk fel, -flekppen. A tbbiesetet is kiszmthatjuk, sszegk adja a megoldst:

    w x3044 A legfeljebb egy tk lehet egy vagy nulla darab. Ez csak kt eset, nincs rtelme ttrni az ellen-tett esemnyre, hiszen ott ht alesetet kellene sszerni. Lssuk ht. Ha nincs tk a leosztsban, akkor a pakliban lev nyolc lapbl nulla darab kerl a ngy lap kz,

    az sszes tbbi piros, zld vagy makk: Ha egy tk van, akkor piros, zld, makk

    lehet hrom: A megolds a kett sszege:

    w x3045 Gondoljunk t rszletesen egy esetet, pldul a hrom tallatot. Hrmasunk gy lehet, ha az tnyerszmbl megjelltnk (kivlasztottunk) hrmat, a maradk kt tippnknek viszont a nemnyer 85 szmbl kell kikerlnie. Ez alapjn minden esetet szmba vehetnk. Ha nincs tallatunk,akkor minden megjellt szmunk a nem nyert-ek kzl kerl ki. Ha pedig telitallatunk van,akkor ott minden megjellt tipp a nyerszmok kzl kerl ki.

    a) b) c)

    d) e) f )

    Figyeljk meg, hogy az sszes lehetsges szmtsk darabszma ami a fentiek

    sszege. Ez azrt lehet, mert az sszes esetet egymst kizr alesetekre bontottuk.

    905

    43949268 = ,

    55

    850

    1

    = .

    54

    851

    425

    = ;

    53

    852

    35700

    = ;

    52

    853

    987700

    = ;

    51

    854

    10123925

    = ;

    50

    855

    32801517

    = ;

    80

    244

    81

    243

    26818

    + = .

    81

    243

    16192

    = .

    80

    244

    10626

    = .

    41

    283

    42

    282

    43

    281

    44

    280

    + + + = 15485.

    43

    281

    324

    284

    15485

    .=

    52

    3 407 6 10

    416!

    ( !) !, .

    523

    493

    463

    433

    .

    324

    214

    29975

    .=

    111

    213

    14630

    = .

    313

    4495 = .

    matematika_11_fgy_mo.qxd 2010.11.16. 9:07 Page 9

  • MEGOLDSOK 11 . VFOLYAM

    10

    w x3046 Egy kzfogs ltrejtthez kt ember kell. A krdst mdosthatjuk gy is: hny fbl vlaszthatunk ki

    sszesen 45-flekpp kettt? Vagy: hatrozzuk meg n termszetes szmot, amire Ez nem

    bonyolult, csak rjuk fel n alatt k defincijt, s egyszerstsnk le (n 2)!-sal:

    Utbbi egyenlsget tekintsk nll egyenletnek s tntessk el a trteket, bontsuk fel a zrjelet,majd rendezzk egy oldalra. gy az n2 n 90 = 0 msodfok egyenletet kapjuk, aminek meg-oldsai: n1 = 10 s n2 = 9. Neknk a feladat szvege szerint csak a termszetes szmok jhetnekszba, gy a trgyalson 10 f volt jelen.

    w x3047 Csak ki kell fejtennk a bal oldalon ll kifejezseket, majd kzs nevezre hozzuk ket:

    A vgn kiemeltnk (n 1)!-t, s sszevontunk a zrjelben. gy ppen (n 1)! n = n! formtkapjuk, s kszen is vagyunk.

    w x3048 a) Az sszefggshez egy soron bell kell hromegymst kvet szmot tallni (vagyis leg-albb a msodik sorbl lsd az brn),pldul:

    1 + 2 2 + 1 = 6,10 + 2 10 + 5 = 35.

    A kapott rtkek kt sorral lejjebb tallhatk(innen az n + 2), s mivel a soron bell azazonos sorszm tagok ferdn balra dlve kvetik egymst (piros vonal), gy az sszeadandkkzl az utols sorszmval egyezik meg az eredmny (innen a k + 1).

    b) A bizonytst elvgezhetnnk a definci alapjn is, azonbanhosszadalmas s nem jelent jat az elz feladathoz kpest.Ezrt prblkozzunk magval az ott trgyalt sszefggssel:

    rdemes az brn is kvetni a levezetst.

    w x3049 a) A feladat az elz pldhoz nagyon hasonlt, csak itt ngy darab egy sorban, egyms utn llszmot kell sszeadnunk (ezrt aztn leghamarabb a harmadik sorban tekinthetjk). Ott pedig

    1 + 3 3 + 3 3 + 1 = 20 vagy ksbb 7 + 3 21 + 3 35 + 35 = 210.1

    2

    6

    20

    70

    252

    1

    3

    10

    35

    126

    1

    4

    15

    56

    210

    1

    5

    21

    84

    1

    6

    28

    120

    1

    7

    36

    1

    8

    45

    1

    9

    1

    10

    1

    1

    1

    3

    10

    35

    126

    1

    4

    15

    56

    210

    1

    5

    21

    84

    1

    6

    28

    120

    1

    7

    36

    1

    8

    45

    1

    9

    1

    10

    1

    1

    nk

    nk

    nk

    nk

    nk

    nk

    nk 1 1

    1 11

    21

    + + + + =

    + + ++

    = ++

    .

    1

    2

    6

    1

    3

    1

    4

    1

    1

    1

    3

    1

    4

    1

    1

    1

    2

    6

    20

    1

    3

    10

    35

    1

    4

    15

    1

    5

    21

    1

    6

    1

    7

    1

    1

    1

    3

    10

    35

    1

    4

    15

    1

    5

    21

    1

    6

    1

    7

    1

    1

    nk

    nk

    n

    k n k

    n

    k n k

    ( )!( ) ( )!

    ( )!( )!

    11

    1 11

    11

    + =!

    +!

    =(( ) ( ) ( )

    ( )!.

    n k n n k

    k n knk

    1 1! + !!

    =

    n

    n

    n n!

    ( )!

    ( ).

    2 2

    1

    245

    != =

    n2

    45 = .

    matematika_11_fgy_mo.qxd 2010.11.16. 9:07 Page 10

  • KOMBINATORIKA , GRFOK

    11

    b) A 20 s 210 rtkeket hrom sorral lejjebb, az utols rtkkel azonos sorszm helyen talljuk.Azaz a sejtsnk:

    c) Az igazolshoz hasznljuk fel htszer az sszefggst. A 3-as szorz

    szksges, ugyanis ha csak ktszer vennnk a kzps elemeket, akkor pldul: 7 + 21 = 28;21 + 35 = 56; 35 + 35 = 70. Ezen hrom szmbl csak az egyik oldalon folytathatnnk, hiszen56 egyszer szerepel. A kzps elemet ezrt meg kell duplznunk, amihez hrom darab 21s hrom darab 35 kell.Megjegyzs: Az elz kt feladatot termszetesen lehet ltalnostani, bvtve a szmok sort.rdemes szrevennnk, hogy az egytthatk is a Pascal-hromszgbl valk. Azonban mostrszletesebben egy tovbbi ltalnostsra hvjuk fel a figyelmet. rjuk le az els sorokat, aholaz sszefggsek elfordulhatnak gy:

    ltalnostva megfigyelsnket, j sejtshez juthatunk. Prbljuk meg bebizonytani, hogy

    w x3050 a) Az sszefggs az egyms utni sorokban ugyanazon sorszm helyeken ll szmok sszege,mindig a sorvgi szls rtktl kezdve. Pldul:

    1 + 3 + 6 + 10 = 20 vagy 1 + 5 + 15 + 35 + 70 = 126.

    b) gy tnik, az sszeg mindig az eggyel lejjebb lev sor eggyel htrbb lev eleme. Sejtsnk:

    c) A bizonytst elvgezhetjk rgztett k-ra n szerinti teljes indukcival, de bevethetnk egy aprtrkkt is. Igaz ugyanis, hogy

    gy az els kttag sszeg egybeolvad, majd a kvetkez kett jra s gy tovbb. Nem tesznk

    mst, mint jra s jra felhasznljuk az sszefggst. Tulajdonkppen

    egy teleszkopikus sszeghez jutunk.

    nk

    nk

    nk

    11

    1

    + =

    kk

    kk

    kk

    kk

    kk

    =

    ++

    ahonnan ++

    + + = ++

    11

    11

    1 21

    , .

    kk

    kk

    kk

    nk

    nk

    nk

    +

    + + + + + + = ++

    1 2 1 11

    .

    1

    2

    6

    20

    70

    252

    1

    3

    10

    35

    126

    1

    4

    15

    56

    210

    1

    5

    21

    84

    1

    6

    28

    120

    1

    7

    36

    1

    8

    45

    1

    9

    1

    10

    1

    1

    1

    3

    10

    35

    126

    1

    4

    15

    56

    210

    1

    5

    21

    84

    1

    6

    28

    120

    1

    7

    36

    1

    8

    45

    1

    9

    1

    10

    1

    1

    n n n nn

    nn0 1 2

    22 2 2 2

    + + + + = .

    1 2 1 20

    21

    22

    42

    1 3 3 1 30

    2 2 22 2 2

    2 2 2 2+ + = + + = s + + + =

    2 2 2 231

    32

    33

    63

    + + + = .

    nk

    nk

    nk

    11

    1

    + =

    nk

    nk

    nk

    nk

    nk

    .2 1

    31

    31

    + 3 + + + =

    ++

    matematika_11_fgy_mo.qxd 2010.11.16. 9:07 Page 11

  • MEGOLDSOK 11 . VFOLYAM

    12

    d) Az sszeg k = 1-re a kvetkez alakot lti:

    vagy ugyanez mskpp rva:

    Ez pedig nem ms, mint az els n termszetes szm sszegre vonatkoz sszefggs.Megjegyzs: Jtsszunk mg az sszefggssel. Hasznljuk ki a Pascal-hromszg szimmetrijt,

    s tkrzzk a szimmetriatengelyre a szerepl tagokat. Felhasznlva az ssze-fggst, a kvetkez alakhoz jutunk:

    A c) rszben lert trkkel ezt is igazolhatjuk, ha felhasznljuk, hogy

    w x3051 A krds kicsit msknt fogalmazva: oldjuk meg a egyenletet. Ezt prbval vagy

    a definci alapjn tudjuk megtenni. Lssuk a tudomnyosabb mdszert. Eltntetve a trtet:

    A jobb oldalon egy egsz szmokbl ll szorzat st kt szorzat szerepel. Radsul mindkett1-tl kezdve halad az egyms utni egszeken, gy tartalmaz sorozatban egyforma tnyezket is.Teht vrhatan bizonyos rtkek dupln szerepelnek a 8 709 120 szorzatt bontsban. Lssuka prmtnyezs felbontst:

    8 709 120 = 210 35 5 7.A 7 s 5 prmtnyez biztosan egyszer szerepel, gy azok csak a hosszabb szorzat ellltsbanvehetnek rszt. A kztk lev 6 is egyszer szerepel, ellltshoz egy 2 s egy 3 prmtnyezszksges. Az 5 eltt kell lennie az 1 2 3 4 szorzatnak, ami elvisz hrom 2 s egy 3 prmtnyezt.Eddig elfogyasztottunk ngy darab 2, kett darab 3, egy 5 s egy 7 prmet. Maradt 26 33. A hrom3 prmbl nem tehetnk kettt a rvidebb szorzatba, mert akkor el kellene jutnunk 6-ig. gy kettmarad a hosszabban, azaz ott lesz 9 is s gy 8 is. Ezzel jabb prmeket hasznlunk el, marad 23 3.Ez pedig pont elg az 1 2 3 4 szorzathoz. Vagyis 8 709 120 = 4! 9! s gy k = 4 vagy k = 9.Teht 13 fbl 4 vagy 9 ft tudunk 715-flekppen kivlasztani.

    w x3052 A krds tfogalmazva: oldjuk meg a termszetes szmokon az egyenletet! Ennek is

    nekilthatunk prblgatssal, m abbl nem sokat tanulunk. Az elz pldhoz hasonlan kezeljka krdst:

    Rutinosan prmtnyezkre bonthatjuk a jobb oldalt (rgtn vlasszuk le a vgrl a 100 = 22 52-t):390700 800 = 28 33 52 7 17 19.

    n

    n

    n

    n

    !

    ( )!,

    !

    ( )!.

    7 777520

    7390700800

    !=

    =

    n7

    77520 =

    1313

    715

    8709120 13

    !( )!

    ,

    ( )!

    k k

    k k

    !=

    = !

    13 715k =

    k k0

    10

    =

    + .

    k k k nn k

    nn k

    nn k0

    11

    22

    11

    1

    + + + + + + + = + .

    nk

    nn k

    =

    1 2 31

    + + + + =+

    2 n n n( ) .

    11

    21

    31 1

    12

    + + + + =

    +n n ,

    matematika_11_fgy_mo.qxd 2010.11.16. 9:07 Page 12

  • KOMBINATORIKA , GRFOK

    13

    Figyeljk meg a msik oldalt is, itt n-tl lefel ht darab egyms utn kvetkez egyre kisebb szmszorzata szerepel. Mivel a 17 s 19 prmek, gy valsznleg szerepelnek a szorzatban. Kztktalljuk a 18 = 2 32-t is. Vrhatan 7-ig nem jut el 19-tl a szorzat, ezrt 14 = 2 7-et kellkeresnnk vagy 21 = 3 7-et. Brmelyik is szerepel, a szorzat szln kell llnia. Ugyanis van kt5-s prmtnyez is, egyik lehet a 15 = 3 5 ptkve, msik pedig a 20 = 22 5-.Gyjtsk ssze a felbontsbl eddig elhasznlt prmeket:

    20 19 18 17 16 15 = 27 33 52 17 19.Maradt mg egy darab 2 s egy darab 7, teht a megolds:

    390 700 800 = 20 19 18 17 16 15 14.Vagyis 20 fbl vlaszthatunk 77 520-flekpp 7-et. Msik megolds nincs.

    w x3053 A feladatra azonnal adhatunk kt magtl rtetd megoldst: a 8568. sor 1. eleme s a 8568. sor8567. eleme is 8568. Igen m, de tallhat-e ms lelhelye a Pascal-hromszgben a jelzettszmnak?Most is fogalmazzuk meg msknt a krdst: mely n s k (n k 0) termszetes szmra teljesl,

    hogy

    Az elz pontban az megoldst adtuk meg.

    Prblgatssal most nem sokra megynk, mert sok az ismeretlen. Vegyk ht a defincit, bontsukfel a 8568-at prmekre, s gondolkodjunk:

    A bal oldalt tekintsk gy, hogy a tbb tnyezbl ll szorzattal leegyszerstettnk, legyen ezmost az (n k)! (A szimmetria miatt nem fontos, melyiket tekintjk.) A tloldalon van egy 17-es prmtnyez, ami elg nagy ahhoz, hogy ne egyszerstett alakblkapjuk. Feltehetjk, hogy a 17 szerepel a bal oldal szmlljban. Nzzk a 17 krli szmokat,pldul az elz feladathoz hasonlan 18 = 2 32-t is ki tudjuk rakni a prmekbl s 14 = 2 7-et is.19 nem szerepel, gy biztosan tudjuk, hogy legfeljebb n = 18. 13 sem szerepel, teht vele mregyszerstettnk. A 18 s 14 kztt lennie kellett 15 = 3 5 s 16 = 24-nek. Hat darab 2-esprmnk azonban nincs a felbontsban ez azrt lehet, mert a szorzatot egyszerstettk k!-sal.Most nzzk meg azt, mi hinyzik a teljes 14 15 16 17 18 szorzatbl, ennek rulkodnia kellaz oszt k!-rl:

    14 15 16 17 18 = 26 33 5 7 17 = (23 32 7 17) (23 3 5) == (23 32 7 17) (2 3 22 5) = (23 32 7 17) (5!).

    Elkszltnk ht: k = 5 s n = 18 megoldst ad. Ugyangy a szimmetria miatt megolds k = 13 is.

    Binomilis egytthatk, ismtlses kombinci megoldsok

    w x3054 a) 9 6b + b2;b) e3 + 6e2f + 12ef 2 + 8f 3;c) x4 8x3 + 24x2 32x + 16;d) 32x5 + 80x4 + 80x3 + 40x2 + 10x + 1;e) 1 3,5a + 5,25a2 4,375a3 + 2,1875a4 0,65625a5 + 0,109375a6 0,0078125a7.

    n

    k n k

    !

    ( )!.

    !=

    2 3 7 173 2

    85681

    85688567

    8568

    = =

    nk = 8568?

    matematika_11_fgy_mo.qxd 2010.11.16. 9:07 Page 13

  • MEGOLDSOK 11 . VFOLYAM

    14

    w x3055 a) x12 + 12x11 + 60x10 + 160x9 + 240x8 + 192x7 + 64x6;

    b)

    c)

    d)

    e) 64x6 576x8 + 2160x10 4320x12 + 4860x14 2916x16 + 729x18.

    w x3056 a) 1 2 sin x cos x; b) 4 sin2x cos2x + 4 sin x cos x + 1.

    w x3057 a) (2a + b)3; b) (0,5x2 2y)3; c) (x2 x3)4; d)w x3058 a) 2; b) 4; c) 32; d) 1.

    w x3059 a) res halmaz. b) 1. c) Nincs ilyen halmaz. d) 4.e) 8. f ) Nincs ilyen halmaz.

    w x3060 Minden bit rtke 0 vagy 1 lehet. Mivel a shortint s az integer egsz szmok, ezrt lehetneknegatvok is: kell hagynunk az eljelnek egy bitet (az elst). A szm rtknek rgztshez gy 7,illetve 15 bit marad. A legnagyobb rtk ezrt a) shortint esetn: 27 = 128; b) integer esetn: 215 = 32 768.A rszhalmazok szmt csak a halmazban lev elemek szma hatrozza meg, mrpedig shortinttpus szmbl 2 128 + 1 = 257 darab lehet, integerbl pedig 2 32 768 + 1 = 65 537. A vlasz:c) 2257; d) 265537.

    w x3061 A kt megadott szm a lnyok s a fik rszhalmazainak szma. Vagyis krds, hogy 2-nek melyikhatvnya az, aminek rtke 1024. Hamar kitalljuk, 210 = 1024. A fiknl ugyanez 213 = 8192.Ebbl addik, hogy a lnyok tzen, a fik tizenhrman vannak az osztlyban, teht az osztlyltszmsszesen huszonhrom. Teht 223-flekppen vlaszthat ki egy csoport az egsz osztlybl.Megjegyzs: Hamarosan megtanuljuk ltalnosan is megoldani a 2x = 1024, n. exponencilisegyenleteket.

    w x3062 Kpzeljk el a csapat sszes tagjt egyms mellett llva a bemutatskor. A bellk kpezhet rszhal-mazokat jellhetjk gy is, hogy egy mnuszjelet kpzelnk afl, aki nem tagja; s egy plusz jeletafl, aki tagja a rszhalmaznak. Mind a tizenegy jtkos felett vagy plusz, vagy mnusz jel llhat,gy 211 lehetsg van klnbz rszhalmazok kpzsre.a) Most ha az Enik feletti jelet mnusznak rgztjk, akkor csak a tbbiek jele vltozhat, teht

    a megolds 210.b) Az elz rszhalmazokat gy kaptuk, hogy Enik jelt mnusznak vettk. Hagyjuk ht ezt gy,

    s most legyen vike jele plusz. Ezt is rgztettk, szabadon vlaszthat marad kilenc f, a meg-olds 29.

    c) Ebben a krdsben t ft kell rgztennk (az mindegy, hogy plusz- vagy mnuszjelet rgztnka jtkos felett kpzeletben). Szabadon hat ft vlaszthatunk, a kpezhet rszhalmazok szma26, ami ppen 25 = 32-ed rsze az sszes lehetsgnek.

    w x3063 a) A fociban semmi sem tiltja, hogy kt vagy tbb szabadrgst ne vgezhessen el ugyanaza jtkos. Mivel ebben az esetben az idbeli sorrendet is figyelembe vesszk, gy 8 f kzlkell kivlasztani az els, a msodik stb. szabadot rg jtkost (ismtlses varici):

    816 2,8 1014.

    x x + .4( )

    x x x x x x x x x x x x7 6 6 5 5 4 4 321 189 945 2835 5103 5103 2187+ + + + + + + ;

    x x x x x x x x x x x

    x

    3 3 3 2 4 3 5 5 3 2 6 3 7

    7

    9 36 84 126 126 84

    36

    + + + + + + +

    +

    ( ) ( ) xx x x x3

    2 8 3 99( ) + + ;

    256 512 448 224 70 1472

    12

    8 7 6 2 5 3 4 4 3 52

    2 63

    7a a b a b a b a b a b a b ab + + + +1128

    8b ;

    matematika_11_fgy_mo.qxd 2010.11.16. 9:07 Page 14

  • KOMBINATORIKA , GRFOK

    15

    b) A sorrend most nem szmt, ezrt ismtlses kombincit kell szmolnunk. A tizenegyjtkosbl elhagyva a hrom csatrt, 8 elem 16 tag ismtlses kombinciinak szma:

    c) Ebben a rszfeladatban a csatrokat engedjk szabadrgst lni, de a kapust nem. Megint ismt-lses varicit szmolunk, az eredmny:

    1013.d) A sorrendre val tekintet nlkl ismtlses kombincihoz jutunk:

    Vegyes sszeszmllsi feladatok (kiegszt anyag) megoldsok

    w x3064 a) b) 4! = 24;

    c) n elem k tag ismtls nlkli variciinak szma.

    w x3065 a)

    b) 32 elem 4 tag ismtls nlkli kombinciinak szma.

    w x3066

    w x3067 a) 3!; b) 12!;

    c) d) 3! (6 5 + 4 10 3 + 10 3) = 1080.

    w x3068 a) b)

    w x3069 a) Ktfle megolds is esznkbe juthat. A szmols megoldshoz fejtsk ki a kt oldalt, majdamivel csak lehet, egyszerstsnk, s tntessk el a trteket:

    Megoldsprokat kapunk, ezeket egy tbl-zatba is foglalhatjuk:

    Vagyis pldul ht elembl annyiflekppen tudunk kivlasztani a sorrendre val tekintetnlkl ngyet, mint hrmat. t elembl pedig annyiflekpp tudunk kivlasztani a sorrendreval tekintet nlkl hrmat, mint kettt.

    k 1 2 3 4 5

    n 1 3 5 7 9

    n

    k n k

    n

    k n k

    n k kn k

    !( )!

    !( ) ( )!

    ,

    , .

    !=

    ! +

    + ==

    1 11

    2 1

    95

    85

    84

    70

    .= =

    9

    9 5

    8

    8 55 8 7 6 5 8400

    !

    ( )!

    !

    ( )!;= =

    12

    3 4 53 166320

    !

    !! ;

    ! !=

    308

    7 !

    3232 4

    4 324

    35960!

    ( )!: ! ,= =

    32

    32 4863040

    !

    ( )!;=

    nk

    kn

    n k !

    !( )!

    ,=

    1004

    3921225 = ;

    10 13 113

    497420+ = .

    8 16 116

    245157+ = .

    matematika_11_fgy_mo.qxd 2010.11.16. 9:07 Page 15

  • MEGOLDSOK 11 . VFOLYAM

    16

    b) Az elz esethez hasonl talaktsokkal ebben az esetben(n k + 1) (n k + 2) = (k 1) k

    alakhoz jutunk. A zrjeleket kifejtve, s rendezve az egyenletet:n2 + (3 2k) n + 2 2k = 0.

    Teht n ismeretlenben msodfok egyenletet kaptunk k paramterrel. Felrva s egyszerstvea megoldkpletben szerepl kifejezseket:

    Az sszeadssal n1 = 2k 2. A kivonssalkapott, k-tl fggetlen n2 = 1 eredmnytnem tudjuk rtelmezni. A tblzat a meg-oldsprokkal mr ismers lehet.A kombinatorika nyelvre fordtva ez azt jelenti, hogy pl. hat elembl annyiflekppen lehetngyet kivlasztani a sorrendre val tekintet nlkl, mint kettt.Megjegyzs: A feladat adja az ltalnosts lehetsgt: mely n, k, r megfelel pozitv egsz

    rtkekre igaz, hogy Azonban az eddig alkalmazott mdszerrel vrhatan

    n-ben r-edfok egyenletet kapunk.A fenti eljrs helyett sokkal egyszerbb, ha felhasznljuk, hogy a binomilis egytthatk ptikfel a Pascal-hromszget. A kkkel jellt rtkek adjk az a) rszben feltett krdsre a vlaszt,a zldek pedig a b) rszfeladatra. Indoklskppen elegend a Pascal-hromszg szimmetrijrahivatkoznunk. gy az ltalnostott krdst is meg tudjuk vlaszolni minden lehetsges n, k s rrtkre.

    w x3070 brzoljuk a csapatokat s a kztk lezajl mrkzseket.

    1

    2

    6

    20

    70

    252

    1

    3

    10

    35

    126

    1

    4

    15

    56

    210

    1

    5

    21

    84

    1

    6

    28

    120

    1

    7

    36

    1

    8

    45

    1

    9

    1

    10

    1

    1

    1

    3

    10

    35

    126

    1

    4

    15

    56

    210

    1

    5

    21

    84

    1

    6

    28

    120

    1

    7

    36

    1

    8

    45

    1

    9

    1

    10

    1

    1

    nk

    nk r

    = ?

    k 2 3 4 5 6

    n 2 4 6 8 10

    nk k

    1 22 3 2 1

    2, ( )

    .=

    matematika_11_fgy_mo.qxd 2010.11.16. 9:07 Page 16

  • KOMBINATORIKA , GRFOK

    17

    a) Sorban haladva az els csapat 11 mrkzst jtszik (piros). A msodik mr jtszott az elsvel,gy 10 j mrkzst jtszik (zld). A harmadik mr jtszott az els kettvel (kk) stb.:

    11 + 10 + 9 + + 1 = 66.

    b) A msik brt tekintve minden csapat 11 mrkzst jtszik. Azonban figyelembe kell vennnk,hogy kt csapat csupn egyetlen egyszer tallkozik, mi viszont minden meccset ktszer sz-moltunk (egyszer az egyik, egyszer a msik csapatnl). A helyes eredmny:

    c) Brmely mrkzs ltrejtthez kt csapat szksges. A feladat tfogalmazhat gy: hny-flekpp lehet kivlasztani 12 elembl kettt ismtls nlkl? A vlasz pedig:

    Megjegyzs: Ha a feladatot ltalnostjuk, az a) s b) rszbl addik az els n termszetes szmsszegre korbban mr megismert kplet is:

    w x3071 a) A legalbb kett 8-as jelenthet kettt, hrmat, ngyet vagy tt. Inkbb szmoljuk a komplementeresemnyt, ha nincs vagy egy 8-as van. Az sszes esetek szma (ismtlses varici) 95. Nincs8-as a szmok kztt 85 esetben. Az egyetlen 8-as llhat t helyen, a tbbi helyre a maradknyolc rtk valamelyikt rhatjuk: 5 84. Az eredmny a fenti rtkek klnbsge:

    95 85 5 84 = 5801.b) Most is rdemes ttrni az ellentett esetek sszeszmllsra, azonban itt ismtlses kombi-

    ncikat kell felrnunk (tanknyvben apr bets rsz). Az sszes esetekben kilenc szmbl

    vlasztunk ki ismtlssel tt: Ha nincs kzttk 8-as, akkor mr csak nyolc szmbl

    vlasztunk tt ismtlssel: Ha egy 8-as van a szmok kztt, akkor a tbbi ngyet

    a maradk nyolc rtkbl vlaszthatjuk, szintn ismtlssel: A vgeredmny:

    w x3072 a) A fagylalt tvtele 20 elem 4 tag ismtls nlkli varicija, ezek szma

    A pnz tadsa 11 elem valamely 5, 3, 3 tag ismtlses permutcija, szmukAz eredmny a kett szorzata:

    b) A fagylaltot 20 elem 3 tag ismtls nlkli kombincijaknt kapjuk, szmuk

    A 2, 2, 1 darab rmt ki kell vlasztanunk az 5, 3, 3 darab azonos kzl, ezt -flekpptehetjk meg. Az eredmny:

    203

    52

    32

    31

    .

    52

    32

    31

    , ,

    203

    .

    20 11

    20 4 3 3 5

    !

    ! ! !

    !

    ( ) !.

    11

    3 3 5

    !

    !.

    ! !

    20

    20 4

    !

    ( )!.

    135

    125

    114

    165

    .=

    8 4 14

    + .

    8 5 15

    + .

    9 5 15

    + .

    1 21

    2+ + + =

    + n n n( ) .

    122

    66 = .

    11 12

    266

    = .

    matematika_11_fgy_mo.qxd 2010.11.16. 9:07 Page 17

  • MEGOLDSOK 11 . VFOLYAM

    18

    c) A fagylaltot 20 elem 5 tag ismtlses varicijaknt kapjuk, sszesen 205-flekppen.A fizets az elz rszkrdshez hasonlan zajlik. Az eredmny ekkor:

    d) Utoljra 20 elem egy 7 tag ismtlses kombincijt kell tekintennk, erre kln-bz lehetsgnk van. Az rmk kivlasztsa most is -flekpp trtnik, azonban mivel egyesvel sz-

    moljuk le ket, sorrendjk is szmt (6 elem 2, 3, 1 tag ismtlses permutcija). Az eredmny:

    w x3073 a) Az els jtkos az eredeti 52 lapbl kap 5-t, ezt -flekpp adhatja neki az oszt. A msodik

    jtkosnak mr csak a maradk 47 lapbl jut 7 darab, -flekpp. Ezt kveten az els

    jtkos -flekpp dobhat 2 lapot a kezben lev 5-bl. A msodik jtkosnak a dobsra

    lehetsge van. A ledobott hat lapot ezutn 6!-flekpp tehetik egyenes sorba. Az eredmnya fentiek szorzata, hiszen egymstl nem fgg esemnyekrl van sz:

    b) A kr alak elrendezs esetn csak a feladat vge vltozik:

    Hatszor tbb lehetsgnk van a jtkban egyenes sorban elhelyezni a lapokat, mint kr alakban.

    w x3074 a) A lapok leosztsakor most sem vehetjk figyelembe a sorrendet, gy a lehetsges leosztsokszma:

    b) Az elbbihez hasonlan:

    c) Fejtsk ki elsnek az a) rszt. Kt helyen is egyszersthetnk:

    A b) esetben is hasonlan vgezhetnk egyszerstseket:

    Mindkt esetben megfigyelhetjk, hogy a nevezben lev faktorilist kijell szmok sszege(pl. 4 + 6 + 8 + 34 = 52) kiadja a szmllban szerepl szmot. Ilyet korbban az ismtlsespermutcinl lttunk (52 elem 4, 6, 8 s 34 tag permutciinak szma ennyi).

    n

    k n k

    n k

    k n k k

    n k k k

    kr

    r

    !( )!

    ( )!( )!

    ( )!1 1

    1

    2 1 2

    1 2 1

    ! !

    !!=

    =! ! !

    ( )!!

    ( )!.

    n k k kn

    k k k n k k k

    r

    r r

    1 2

    1 2 1 2

    524

    486

    428

    524 48

    486 42

    428 34

    524

    =! ! !

    =!

    !!

    !!

    !!

    !! ! ! 6 8 34!.

    nk

    n kk

    n k kk

    n k k kk

    r

    r1

    1

    2

    1 2

    3

    1 2 1

    .

    524

    486

    428

    .

    525

    477

    52

    74

    5

    !

    525

    477

    52

    74

    6

    !

    74

    52

    477

    525

    20 7 17

    52

    33

    31

    62 3 1

    +! !

    !!.

    52

    33

    31

    , ,

    20 7 17

    +

    20 53

    32

    32

    5 .

    matematika_11_fgy_mo.qxd 2010.11.16. 9:07 Page 18

  • KOMBINATORIKA , GRFOK

    19

    GRFOK pontok, lek, fokszm megoldsok

    w x3075 A keresett grf az brn lthat.

    w x3076 A keresett grf az brn lthat.

    w x3077 MJ, MG, JH, JG, NG.

    w x3078 A keresett grf az brn lthat.

    w x3079 a) 263; b) 536 4.

    w x3080 a) A keresett grf az brn lthat.b) Gbor s Zoli.

    w x3081 A keresett grf az brn lthat.

    w x3082 a) Pldul: buszfordul, zskutca.b) Egy utcrl laktelepre vezet t kt bejrattal.

    w x3083 Nem.

    w x3084 a) A keresett ngy grf az brn lthat.

    A

    L

    F

    CS

    A J

    E

    CSZ

    G

    C

    T

    007 008

    009

    LB CS

    U

    A

    T

    B

    GK

    A

    matematika_11_fgy_mo.qxd 2010.11.16. 9:07 Page 19

  • MEGOLDSOK 11 . VFOLYAM

    20

    b) A keresett nyolc grf az brn lthat.

    w x3085 A: 5, H: 4, G: 2, N: 4, J: 2, M: 3.

    w x3086 A keresett grfok az brn lthatk.a) 4;b) 3;c) 3.

    w x3087 Hrom lehetsg van:1, 1, 2, 2; 1, 1, 1, 3; 0, 2, 2, 2.

    A keresett grfok az brn lthatk.

    w x3088 Az a) s b) rszfeladat megoldsa az brn lthat.c) Ilyen grf nem ltezik.

    w x3089 b) Vrhatan a ngyfokakbl.

    w x3090 A keresett grfok az brn lthatak.

    w x3091 A keresett betk sorban: a) b) c)

    a)

    1

    4 3

    2

    1

    4

    3

    27

    6

    5

    1

    25

    34

    b) c)

    a) b)

    A

    B C

    A

    B C

    A

    B C

    A

    B C

    A

    B C

    A

    B C

    A

    B C

    A

    B C

    matematika_11_fgy_mo.qxd 2010.11.16. 9:07 Page 20

  • KOMBINATORIKA , GRFOK

    21

    w x3092 A huszr lpsi szablya alapjn az brn lthat helyekre juthat ela megjellt pontokbl. gy a) G8 pont fokszma 3; b) D4 pont fokszma 8. ( )

    w x3093 Tekintsk vgig a figurkat. A kirly nem ilyen, hiszen a sarok-ban (3) kevesebb lehetsge van lpsre, mint a tbla kzepn (9).A vezrnek hasonlan a kirlyhoz, a sarokban llva (21) kevesebba lehetsge, mint a kzps ngy mezben (27). A futnak asarokbl indulva 7, mg a kzps ngy mez valamelyikbl13 lpsi lehetsge van. A bstya azonban a sarokbl (14)ugyanannyi lpst tehet, mint kzpen llva (14). gy tnik,hasonl a helyzet a gyaloggal: az res tbln mindig egyet lphetelre, gy a kztes mezket jell grfpontok fokszma 2 azonban a tbla kiindulpontjrl csakelre lphet egyet, gy ezen pontok fokszma 1. Teht az egyetlen megolds a bstya.

    w x3094 Tudjuk, hogy egy n cscs teljes grfnak le van. Ha egy egyszer grfba mrberajzoltunk n darabot, akkor mg

    lt kell megrajzolni, hogy teljess vljon.

    w x3095 Ha a berajzolhat lek maximlis szmt keressk, akkor tekintsnk csak egyetlen izollt pontota grfban ugyanis minl tbb izollt pontot kpzelnk el, annl kevesebb lt rajzolhatunk meg.A maradk t pont kztt akkor rajzoljuk a legtbb lt, ha teljes grfot ksztnk. Ennek leinekszma pedig

    w x3096 Akik 6-ot mondtak, azok sszesen 7-en vannak egy nemzetbl. Akik 4-et mondtak, azok 5-en.Azonban mivel 12 f mondta a 4-et, legalbb hrom tfs klnbz nemzet tagjai. Aki 2-tvlaszolt, azok hrman vannak. gy mindkt krdst megvlaszolhatjuk:a) t klnbz nemzet tagjai dolgoznak a cgnl.b) sszesen 18 ft krdezett meg a fnk, maradt mg 7 f. 2 fnek kell mg 6-ot, 3 fnek 5-t,

    2 fnek 2-t vlaszolni a cgvezet krdsre.

    w x3097 a) Vigynk rendszert az esetek sszeszmllsba. Vizsgljuk meg, ha 0, aztn 1, majd 2, 3, 4, 5, 6le van az egyszer grfnak. A 11 megolds az brkon lthat.

    rdemes megfigyelni, hogy az egyes esetek szimmetrikusak: 0 l eset annyi van, mint 6;1 l, mint 5; 2 l eset pedig, mint 4. Ez abbl addik, hogy n lt berajzolni pontosan annyi-flekppen tudunk, mint n lt a teljes grfbl letrlni.

    0 l: 1 l:

    4 l: A5 l: 6 l:

    2 l: 3 l:

    B

    CD

    52

    5 5 12

    10

    = =

    ( ).

    n nn

    n n ( )

    ( )1

    2

    3

    2=

    n n n2

    12

    =

    ( )

    1 1

    2 2

    3 3

    4 4

    5 5

    6 6

    7 7

    8 8

    B

    B

    A

    A

    C

    C

    D

    D

    E

    E

    F

    F

    G

    G

    H

    H

    matematika_11_fgy_mo.qxd 2010.11.16. 9:07 Page 21

  • MEGOLDSOK 11 . VFOLYAM

    22

    b) Az brn az res ABCD grfot tekintve hat klnbz hely van. Ezek mindegyikre vagyrajzolunk lt, vagy nem. Ez lenknt kt lehetsg. Teht az egyszer grfok szma 26 = 64.(Ezzel biztostottuk azt is, hogy a keletkez grf valban egyszer lesz.)

    w x3098 A 3097. feladat b) rszt kell tovbbgondolnunk. Tekintve az n pont teljes grf minden egyes lt,azt vagy hozzvesszk a kialaktand egyszer grfunkhoz, vagy nem. Ez minden l esetben kt

    lehetsg, az n pont teljes grf leinek szma pedig A megolds teht

    w x3099 Mivel a feladat egyszer n pont grfokrl szl, ezrt a legnagyobb fokszm pont fokszma leg-feljebb n 1. Teht a feladatban krdezett fokszmok csak n 1, n 2, n 3, , 2, 1, 0 lehetnek.Ez n darab klnbz rtk. Lehetsges-e ilyen grf? Nem, mert az n 1 s a 0 fokszmokkizrjk egymst: ha van n 1 fok pont, akkor nincs 0 fok s fordtva.Megjegyzs: Ugyanezen gondolatmenettel igazolhatjuk, hogy minden egyszer grfban van leg-albb kt azonos fokszm pont. Ha ezt megtettk, a feladat megvlaszolshoz elegend errehivatkoznunk.

    w x3100 brzoljuk a keresett grfot, majd az brrl leolvashatjuk,hogy az els szinten 2 l van, majd minden szinten 2-vel tbb.Ha n pont szerepel az utols sorban, akkor felette 2 (n 1) lttallunk. Azaz

    2 + 4 + 6 + + 2 (n 1) = 2 (1 + 2 + 3 + + n 1) == n (n 1)

    darab lt tallunk a grfban, ha az utols sorban n tgla van.

    w x3101 a) Haladjunk sorban a grf pontjainak szma szerint. Mivel a grf egyszer s van hrom le,legalbb hrom pontjnak is lennie kell. Hrompont, hroml grf egy van. A ngyponts hroml grfokat a 3087. feladatban gyjtttk ssze, hrom darabot talltunk. tpont shroml grfokat gyrthatunk a ngypontakbl, csak vegynk hozzjuk mg egy izolltpontot. Ezeken kvl mg egy lehetsgnk van: ha egy pontprt elklntnk a tbbi hromtl.Hatpont grfokat hasonlan tudunk gyrtani az tpont grfokbl. Itt is csak egy tovbbilehetsg van: ha elklntnk hrom, llel sszekttt pontprt (utols bra).

    gy hat pont esetben sszesen t klnbz hroml egyszer grfot kszthetnk el. Ha mosttovbb szeretnnk nvelni a pontok szmt, hiba: nem lesz tbb lehetsgnk hroml egy-szer grfot ellltani.

    b) Az elz rsz alapjn elmondhatjuk, hogy akkor rjk el a legnagyobb szabadsgi fokotaz lek elhelyezkedsben, ha minden egyes lt klnll grfnak is tekinthetnk. Nem nehzvgiggondolni, hogy ehhez legalbb 2n pontra van szksg.

    w x3102 A feladat szerint nincs kiktve, hogy csak egyszer grfokkal dolgozhatunk, teht a grfban hasz-nlhatunk tbbszrs s hurokleket is. Tudjuk, hogy a fokszmok sszege minden grfban kt-szerese az leknek, ezrt a fokszmok sszegnek pros szmnak kell lennie. Pratlan sok pratlanfok pontot tartalmaz grf nem rajzolhat. Pl. 2, 3, 4 fokszm pontokbl ll grf nemkszthet. St, a feltteleknek megfelel hatpont grf sem: akr pros, akr pratlan szmmalkezdjk a fokszmok felsorolst, mindig hrom darab pratlan szmnak kellene lennie.

    2

    4

    6

    2 2n.n

    2 .

    matematika_11_fgy_mo.qxd 2010.11.16. 9:07 Page 22

  • KOMBINATORIKA , GRFOK

    23

    Az 1, 2, 3 fok pontokbl ll grfot meg tudjuk rajzolni egyvagy kt hurokl segtsgvel. St, ha brmelyik grf mindenegyes pontjra rajzolunk mg k darab huroklt, akkor a pontokfokszmai:

    1 + 2k, 2 + 2k, 3 + 2k.Ilyen grfok teht biztosan kszthetk.Gondolkodjunk tovbb a msodik bra alapjn. Egyrszt pros (2n) fok pontot mindig kszt-hetnk elegend (n) hurokl segtsgvel. Msrszt pratlan (2n + 1) fok pontbl mindig prossoknak kell lennie. lltsunk prba kt ilyen pontot, s kssk ssze ket egy llel, majd illessznkmegfelel szm huroklt a pontokra. gy ppen a megfelel grfot kszthetjk el. sszefoglalva: igen, lehetsges, egsz pontosan mindig rajzolhat olyan grf, melyben a pontokfokszmai egymst kvet pozitv egsz szmok, ha a szmok kztt pros sok pratlan van.

    w x3103 A feladatot lefordtva a grfok nyelvre: Az A, B, C, g, v, pontokat akarjuk gy lekkel sszektni, hogy minden nagybetspont ssze legyen ktve minden kisbets ponttal, de ne legyenekmetsz lek. Bizonyos szm prblkozs utn megsejtjk, hogyez nem lehetsges.A bizonytshoz elszr azt kell szrevennnk, hogy a bektsek sorn mindig ki kell alakulniaegy zrt ngyszgnek. (Ngy darab 2 fok pontnak, melyek egyszer grfot alkotnak a rajzonAvBg.) A harmadik hz vagy a ngyszg belsejbe, vagy kvlre esik. Ebbe is bektve a ktkzmvet, mr hrom zrt ngyszgnk lesz(a rajzon BvCg, AvCg). Mivel a kt lehetsga grfok szempontjbl megegyezik, elg azegyiket tekintennk. Brhogy is tettnk eddig,a harmadik kzm a nagy ngyszgn kvlre,vagy az egyik belsejbe, vagy a msik belsejbe esik. Mindhrom esetben lesz egy olyan ngyszg,amit tekintve a harmadik kzm azon kvl/bell van, azonban valamelyik hz meg bell/kvl.Ezt a kettt gy nem kthetjk ssze korbbi l metszse nlkl (szaggatott lek).Megjegyzs: Br a feladatot grfokkal brzoljuk, igazbl a topolgia tmakrbe tartozik.

    GRFOK t, vonal, sta, kr, Euler-vonal (kiegszt anyag) megoldsok

    w x3104 Pldul: ABE, ABCBDBE.

    w x3105 Pldul: ABFECD, ABFEDC, AFEDCB, ABCDEF, ADCBFE.

    w x3106 DACBA, DABCA, ABCAD, ACBAD.

    w x3107 a) Pldul: ABABDF. b) Pldul: ABCEBDF. c) Pldul: ABDF.

    w x3108 Igen, maga a grf egy kr: ABDFGHECA.

    w x3109 Nincs, a zrt s nyitott Euler-vonal kizrja egymst: a nyitott vonalhoz kt pratlan fok pont szk-sges, zrt vonalhoz viszont minden pont fokszmnak prosnak kell lennie.

    w x3110 Nincs. Ha egy sta nem vonal, akkor legalbb ktszer thalad egy len, gy annak vgpontjain is.

    w x3111 a) Van zrt Euler-vonal, pldul: ABCD. b) Van nyitott Euler-vonal, pldul: BACDBC.c) Nincs.

    A A Ag g g

    v v v

    C C

    B B B

    C

    A g

    v B

    C

    A g

    v

    C

    B

    2

    31

    1

    2 3

    matematika_11_fgy_mo.qxd 2010.11.16. 9:07 Page 23

  • MEGOLDSOK 11 . VFOLYAM

    24

    w x3112 Ahhoz, hogy a ceruza felemelse nlkl lerajzoljuk, az alakzatban nyitott vagy zrt Euler-vonalnakkell lennie. gya) lerajzolhat (nyitott); b) lerajzolhat (nyitott);c) nem rajzolhat le; d) lerajzolhat (nyitott).

    w x3113 A keresett grfok az brkon lthatk.

    w x3114 Az brkon mr az irnyts nlkli grfok lt-hatk. A grfok nem sszefggek, mindegyikkt komponensre esik szt.

    w x3115 A megoldsban kt-kt egyszer felttelt adunk a fokszmok segtsgvel arra, hogy egy grf nemsszefgg, illetve arra, hogy sszefgg.a) Termszetesen a grf nem lehet sszefgg, ha van izollt pontja.

    Ha egy grfnak van 0 fok pontja, akkor nem sszefgg.

    Ha a grf lei elg ritkk, akkor sem lesz sszefgg. Pldul ha 3 vagy tbb pontbl ll grfbanminden pont foka maximum 1, akkor nem alakulhatnak ki lncok, amik sszefznk a pon-tokat. (Ha megengedjk 2 fok pontok ltezst is, akkor mr ltrejhet egy olyan lnc, amelya grf sszes pontjt tartalmazza.) Ha egy n > 2 cscs grfban nincs 1-nl nagyobb fokszm pont, akkor nem sszefgg.

    b) Ha tallunk a grfban egy olyan pontot, amelyhez az sszes tbbi pont kapcsoldik, akkor ezena ponton keresztl brmelyik pontbl brmelyik pontba eljuthatunk.Ha egy n pont grfnak van n 1 fok pontja, akkor sszefgg.

    Ha van olyan pont, amelyhez majdnem minden pont kapcsoldik, akkor a grf sszefgg-sghez elegend, hogy a lemarad pont is kapcsoldjon a tbbihez.Ha egy grfnak van n 2 fok pontja, de nincs 0 fok pontja, akkor sszefgg.

    w x3116 Ha az bra grfjnak kt pontja sszekttetsben van egymssal, az azt jelenti, hogy a ngyzetrcsmegfelel sornak s oszlopnak metszetben lev pixel ki van sznezve. Amennyiben a grfbanegy ponthoz tbb msik kapcsoldik, gy az adott sorban vagy oszlopban tbb pixel tallhat (hogyoszloprl vagy sorrl van sz, az l irnytsa adja meg). Az sszefgg komponensek pontjai soro-kat s oszlopokat jellnek ki. A grf klnbz komponensei teht olyan rszbrkat jellnek, melyek klnbz sorokbans oszlopokban helyezkednek el, vagyis az egyes rszeknek nincs kzs ngyzetrcs-oldaluk.(Ilyen ktkomponens brkat lttunk a 3114. feladatban.)

    w x3117 Elszr is rjuk fel a szvegben szerepl sszefggst jelekkel. Legyen a grf n pont, rendel-kezzen e darab llel, s a minimlis fok pont fokszma legyen f, a maximlis fok pont foka F.Ekkor a feladat lltsa:

    f e

    n

    F

    2 2 .

    a) b) c)1 1

    3

    2

    21

    4

    2

    334

    5

    a) b) c)

    matematika_11_fgy_mo.qxd 2010.11.16. 9:07 Page 24

  • KOMBINATORIKA , GRFOK

    25

    Tntessk el a trteket a kifejezsbl, szorozzuk vgig az egyenltlensget 2-vel s n-nel. gy azn f 2 e n F

    sszefggst kapjuk. Ez termszetesen teljesl, hiszen 2 e a grf fokszmainak sszege. A baloldalon ll n f akkor lenne a fokszmok sszege, ha minden pont foka megegyezne a minimlisfok pont fokszmval (aminl termszetesen lehetnek nagyobb fok pontok is a grfban).Hasonlt mondhatunk a msik egyenltlensgre is, st az egyenlsgek csak egyszerre teljeslhetnek.

    w x3118 Ttelezzk fel az llts ellenttt. Tegyk fel, hogy van olyan grf, amelynek 2n pontja van,s minden pont fokszma legalbb n, de legalbb kt komponensre esik szt. Mivel a grfnak 2ncscsa van, az egyik komponensbe biztosan n vagy annl kevesebb cscs kerl. Mivel a grf egy-szer, gy komponensei is egyszerek. Ellentmondshoz jutunk, ugyanis egy legfeljebb n pontegyszer grfban a legmagasabb fok pont fokszma legfeljebb n 1 lehet, ellenttben a feladatltal emltett n-nel. Vagyis a feltett llts hamis, amibl addik, hogy ellentte igaz.Megjegyzs: A feladat lltst indirekt ton igazoltuk.

    w x3119 Tekintsnk elszr kevesebb pont grfokat. A legkisebb egyszer grfnak, mely tartalmazhat krt,hrom pontja van. A ngy- s tpont egyszer grf pontjai is csak egy krt alkothatnak.A hatpont egyszer grf pontjai alkothatnak egy krt vagy kt hrompont komponenst. Htpontgrf megint vagy egyetlen kr, vagy egy hrom- s egy ngypont komponens. Nyolcpont grfllhat egyetlen krbl, vagy egy hrom- s egy tpont komponensbl, vagy kt ngypont kompo-nensbl. Kilencpont grf llhat hrom hrompont komponensbl, vagy egy ngy- s egy tpontkomponensbl vagy egyetlen krbl. Megfigyelseinkbl kitnik, hogy csak a 3, 4 vagy 5 pont grfokalkothatnak egyetlen mdon krt. A tbb pontbl ll grfokat mr felbonthatjuk ilyen sszetevkre.

    Az ltalnos megoldshoz fogalmazzuk t a krdst: hnyflekppen bonthatjuk n-t 3, 4 vagy 5tbbszrseinek sszegre? Mivel minket csak a lehetsges vlaszok maximuma rdekel, elegenda legrvidebb (hrompont) krk szmt megllaptani. Azt kell kiszmolnunk, hogy n-ben hnyegsz szmszor van meg a 3.

    Pldul az n = 15 pont grfban lehet t darab hrom hossz kr; n = 16 pont grfban lehet ngyhrom- s egy ngypont kr; n = 17 pont grfban lehet ngy hrom- s egy tpont kr.gy mindegyikk legfeljebb t krt tartalmazhat.

    Fagrfok (kiegszt anyag) megoldsok

    w x3120 Mindkettbl egy.

    w x3121 Mivel kt pont kztt pontosan egy t vezet, a grf 13 l fa. gy 14 pontja van.

    w x3122 a) Fa. Van 8 pontja, 7 le, 4 levele, 2 hrom- s 2 ktfok pontja.b) Nem fa, mert krt tartalmaz.c) Fa. Van 9 pontja, 8 le, 5 levele, 3 hrom- s 1 ktfok pontja.d) Nem fa, mert nem sszefgg.

    w x3123 a) Pldul lncnak vagy fonalnak. b) Kett.

    matematika_11_fgy_mo.qxd 2010.11.16. 9:07 Page 25

  • MEGOLDSOK 11 . VFOLYAM

    26

    w x3124 Kt klnbz fagrf ltezik.

    w x3125

    w x3126 A fkat ms mdon is felrhatjuk:a) 2 32 7 13; b) 24 52 11 13. ( )

    w x3127 Az lek szma a fokszmok sszegnek fele,teht 8. Mivel fa, gy eggyel tbb pontja van,mint le: 9. A levelek szma:

    2; 3; ; 8.

    w x3128 a) (8 2) : 2 = 3;b) (1 + 2) (12 : 3) = 12.A c), d) s e) alpontok megoldsa az brnlthat.

    w x3129 A mveletek fagrfok segtsgvel:

    Megjegyzs: A mveleti tulajdonsgokat is lefordthatjuk a grfok nyelvre. Pldul a kommuta-tivits jelentse, hogy a mvelet alatti kt gat felcserljk.

    w x3130 Igen, van. Egyrszt minden fban van legalbb kett elsfok pont, msrszt nyitott Euler-vonalltezsnek szksges felttele pontosan kett pratlan fok pont ltezse. Elbbi megjegyzsbladdan a kett pratlan fok pont csak elsfok lehet. Olyan fa, amelyben pontosan kett els-fok pont van, nem tartalmazhat elgazst, teht csak egyetlen lnc lehet. Megjegyzs: Pontosan ilyen grfokrl volt sz a 3123. feladatban.

    a b

    a a

    b bc c

    c

    ca b a ab

    a = b b a a+( + ) =b c ( + )+a b c

    ab

    a ( + ) =b c ( )+a b ( )a c

    +

    + +

    + +

    +

    c) e)d) +

    2 6 3 4

    1 3 2 4

    + +

    +

    1 2 3

    a) b)

    4

    6 273

    2 3 3 91

    7 13

    1638

    286

    57200

    1432

    1311

    2 2

    10

    2 5 5

    20

    200

    a) b) c)

    ktflekppen hromflekppen tflekppen

    matematika_11_fgy_mo.qxd 2010.11.16. 9:07 Page 26

  • KOMBINATORIKA , GRFOK

    27

    w x3131 a) Furcsa, hogy a grfban nem ismerjk pontosan a mveletek szmt. rjuk t a grf alakot hagyo-mnyos formba:

    2 (2 + (2 (2 + (2 1))))).Fejtsk ki bellrl, s prbljunk meg valamifle szablyszersget tallni.

    2 1 = 1; 2 + 1 = 3; 2 3 = 1; 2 + (1) = 1; 2 1 = 1; stb.Azt ltjuk, hogy ngy lps utn visszajutunkaz els llapothoz. Az sszeg a mveletek sz-mtl (egsz pontosan a mveletek szm-nak nggyel val osztsi maradktl) fggen vltozik (k 0). Az 1 s 3 maradk jelli a ki-vons, a 2 s 0 maradk pedig az sszeads mveletet. Mivel a pratlan maradkok egyszer pozi-tv, egyszer negatv eredmnyt adnak, az sszeg eljele a kivonsok szmnak paritstl fgg.

    b) Tegyk ugyanazt, mint az elbb:2 : (2 (2 : (2 (2 (2 : (2 (2 : 2))))))).

    Bellrl kifejtve a zrjeleket:2 : 2 = 1; 2 1 = 2; 2 : 2 = 1; 2 1 = 2; stb.

    Ebben az esetben az oszts mveletek mindig 1-et adnak eredmnyl, fggetlenl azok szmtl.

    w x3132 Ha egy fagrfnak 15 pontja van s 12 levele, akkor mg van 3 pontja, ami nem levl. Ezen pontokfokszma nagyobb, mint 1, s egymshoz kapcsoldnak. Hrom pont csak gy alkothat ft, ha kr-mentesen egymshoz kapcsoldnak, fokszmaik a leveleket nem tekintve 1, 2, 1. A levelek hozz-juk kapcsoldnak valamilyen mdon, de a kt szls ponthoz legalbb egy-egy (a kzps ponthoznem felttlenl kell, hogy levl kapcsoldjon). Ezek utn megvlaszolhatjuk a krdst.a) A legkisebb fok pont fokszma 2, a legnagyobb 12.b) A grf leghosszabb tja egy levltl indul, thalad a hrom kzbls ponton, majd egy msik

    levlen vgzdik. Ez brmely, a feltteleknek megfelel fa esetben gy van. Az t hossza ngy l.

    w x3133 Keressnk szlssges eseteket! Ilyen eset pldul, ha az t komponensbl egy tartalmazzaaz sszes lt (16 l, 17 pont), a msik ngy komponens pedig egy-egy izollt pontbl ll. Ekkora grf sszesen 17 + 4 = 21 pontot tartalmaz.Ha kicsit vltoztatunk az erdn, s eggyel nveljk az egyik komponens leinek szmt, akkorez a komponens mr eggyel tbb pontot tartalmaz, a legnagyobb komponens ezzel szembeneggyel kevesebbet. Teht az sszpontszm nem vltozik. A fenti gondolatot ltalnosthatjuk.Jelljk a, b, c, d, e egsz szmok az egyes komponensek leinek szmt (0 a; b, c, d, e 16).Ekkor:

    a + b + c + d + e = 16,az egyes komponensek pontjainak szma pedig rendre:

    a + 1, b + 1, c + 1, d + 1, e + 1,hiszen minden komponens fagrf.Az sszes pontok szma pedig valban fggetlen a, b, c, d, e-tl:

    (a + 1) + (b + 1) + (c + 1) + (d + 1) + (e + 1) == a + b + c + d + e + 5 = 16 + 5 = 21.

    w x3134 A grfban a pontok fokszmainak sszege 2n. Ebbl addik, hogy leinek szma n.a) Elszr ttelezzk fel, hogy a feladatban emltett grf sszefgg. Mivel grfunknak n pontja

    s n le van, gy nem lehet krmentes: az n pont fnak, ami maximlis krmentes grf, csakn 1 le van. Mivel itt az lek szma eggyel tbb, mr nem lehet krmentes, kvetkezskppena grfban van kr.

    Mvelet szma 4k+ 1 4k+ 2 4k+ 3 4 (k+ 1)Eredmny 1 3 1 1

    matematika_11_fgy_mo.qxd 2010.11.16. 9:07 Page 27

  • MEGOLDSOK 11 . VFOLYAM

    28

    Amennyiben a grf nem sszefgg, azaz tbb komponensbl ll, akkor valamelyik sszefggkomponensben legalbb annyi pontnak kell lenni, mint amennyi le van. (Ugyanis ha mindenkomponensben kevesebb l lenne, mint amennyi pont, akkor az lek s pontok sszege nemlenne egyenl.) Erre a komponensre pedig alkalmazhatjuk az elz bekezds gondolatmenett.Ezzel bizonytottuk, hogy az n pont, n l grfban van kr.

    b) Ttelezzk most fel, hogy az n pont, n l grf egyszer, sszefgg s legalbb kett krttartalmaz. Ekkor kt eset lehetsges: vagy van kt klnll kr a krk kztt, melyeknekminden le klnbz; vagy van kt kr, melyeknek van legalbb egy kzs le.Ha az els lehetsg ll fenn, akkor valamelyik krbl egy lt trlve a kapott grf mg mindigsszefgg, n pont, viszont n 1 le van. Mivel tbb klnll krt tartalmazott, s csakaz egyik kr egyik lt trltk, gy mg mindig tartalmaz krt. Ez azonban nem lehetsges,mert az n pont, n 1 l egyszer, sszefgg grf fa, vagyis krmentes. Ebben az esetbenellentmondsra jutottunk.Ha van kt kr, melyeknek van kzs le, akkor ezt az lttrlve, ismt n pont, n 1 l, egyszer, sszefgg grf-hoz jutunk. Azonban a feltevs alapjn ez a grf is tartalmazkrt, ugyanis a trlt l kt vgpontja kztt kt klnbzt vezet: egyik t az egyik kr visszamaradt lein t, msik ta msik kr visszamaradt lein t. (Amennyiben nem a kzslek kzl trlnk egyet, akkor az egyik kr megmaraderedeti llapotban, vagyis visszakapjuk az els lehetsget.)Teht ebben az esetben is ellentmondsra vezetett a gon-dolatmenet.Brhogyan is okoskodtunk, feltevsnk mindig ellentmondsra vezetett. Teht az ellentte kellhogy igaz legyen: az n pont, n l, egyszer s sszefgg grf pontosan egy krt tartalmaz.(Emlkeztetl: az a) pontban bizonytottuk, hogy az n pont, n l grf mindig tartalmaz krt.)

    Megjegyzs: A feladat b) rszben indirekt bizonytst alkalmaztunk.

    w x3135 a) Ttelezzk fel, hogy p darab (p nemnegatv egsz) legalbb hromfok pontja van a fnak.Ezen pontok fokszmainak sszege ekkor legalbb 3n. Elsnek trljk a grfbl a ktfok pontokat gy, hogy a pontot elhagyva szomszdait egyet-len llel ktjk ssze. Ezekrl egyrszt amgy sem szl a feladat, msrszt csupn annyiszerepk van, hogy nyjtjk a grfot, az elgazsokat s a leveleket ktik ssze. A grf a trlsutn is fa, hiszen egyszer maradt, sszefgg s krmentes. Fontos megjegyeznnk, hogya megmarad pontok fokszmt nem vltoztatja meg a ktfok pontok trlse.Ezutn trljk a grfbl a leveleket is, a hozzjuk kapcsold lekkel egytt. A grf a levelektrlse utn is fa marad. A megmarad pontok pontosan a p darab, eredetileg legalbb harmad-fok pontok lesznek. Mivel ft alkotnak, sszekt leik szma p 1. Fokszmaik sszege ezrt

    2 (p 1) = 2p 2.Mivel eredetileg a fokszmsszeg legalbb 3p volt, gy legalbb p + 2 darab l hinyzik. Mivela ktfok pontok nem vltoztattak a tbbi pont fokszmn, gy a hinyz fokszmot csakaz elsfok pontok ptolhatjk, mgpedig egy fokszmot egy pont. Legalbb hromfok pontpontosan p darab volt, elsfok pont pedig legalbb p + 2, gy az elsfok pontok szmalegalbb kettvel tbb a legalbb hromfok pontok szmnl.

    b) Az a) rsz gondolatmenetbl vilgos, hogy akkor van pontosan 2 elsfok ponttal tbb a grf-ban, mint legalbb hromfok pont, ha utbbiak fokszmsszege pontosan 3p. Ez pedig csakakkor lehetsges, ha minden legalbb hromfok pont foka pontosan hrom, nincs negyed vagymagasabb fok pont a fagrfban.

    matematika_11_fgy_mo.qxd 2010.11.16. 9:07 Page 28

  • KOMBINATORIKA , GRFOK

    29

    A kombinatorika gyakorlati alkalmazsai megoldsokw x3136 a) A felttelek szerint az els szoba egyik gpe ssze van ktve

    a msik kt szoba egy-egy gpvel. Mivel a msik kettszobban is pontosan egy gp van sszekttetsben a tbbiszobval, gy ezek a gpek mr meghatrozottak. A gpeketnem klnbztetjk meg. A hlzat az brn lthat.

    b) Ebben a hlzatban van hrom darab tpont teljes grf (ssze-sen tizent pont), illetve az ezeket sszekt hrom l. Tehta 15 ponthoz kapcsoldik sszesen 3 10 + 3 = 33 l. Vilgos,hogy a grf sszefgg, gy leinek elhagysval kszthetbelle fa. Azonban egy 15 pont fban maximum 14 l lehet,gy grfunkbl el kell hagyni pontosan 33 14 = 19 lt.

    c) Az eredeti felllshoz kpest annyi vltozott, hogy most nem felttlenl ugyanaz a gp tartjaa kapcsolatot a msik kt szobval. Azaz az els szoba t gpe kzl valamelyik kapcsolatbanll a msodik szoba t gpnek valamelyikvel. Ez 5 5 = 25 lehetsg. Ettl fggetlenl ugyaneza helyzet a msodik s harmadik, illetve a harmadik s els szoba kztt is. A lehetsgek szmaa helyisgek kztti kapcsolatok kiptsre sszesen:

    d) Mivel minden helyisgbl kett gpet vlasztanak ki, az sszes lehetsgek szma a hrom fg-getlen ismtls nlkli kombinci szorzata:

    e) Termszetesen hrom kbellel nem lehet minden gpet hlzatba ktni. Legfeljebb ngyet lehetsorba vagy csillag alakzatba ktni, illetve a msik vglet, hogy kett-kett-kett darabot hromcsoportba (a szveg szerint felhasznljuk mind a hrom kbelt). Ttelezzk fel, hogy a hat gp 1-tl 6-ig van sorszmozva.

    Ekkor kzttk lehetsg van kbel elhelyezsre.

    Ebbl a 15 helybl vlaszthatunk ki hrmat, ahova tnyle-gesen kbelt helyeznk. gy a hrom kbellel sszesen

    lehetsg van a hat szmtgp sszekapcsolsra, ha azokat megklnbztetjk. Az brnszrkvel a lehetsgek, pirossal egy konkrt megvalsts lthat.

    f ) Ha nem klnbztetjk meg a gpeket, akkor a hrom kbel elhelyezsre sszesen t lehet-sgnk addik. Az brkon lthat, hogy az izollt pontok szma 0 s 3 kztt vltozik.

    623

    153

    455

    = =

    62

    15 =

    52

    52

    52

    52

    10 10003

    3

    = = = .

    5 5 1562523 6( ) = = .

    matematika_11_fgy_mo.qxd 2010.11.16. 9:07 Page 29

  • MEGOLDSOK 11 . VFOLYAM

    30

    w x3137 a) Nincs.b) Unoktl ddszlig hat generci (unokagyereksajt magaszlnagyszlddszl).c) Lssuk generci szerint: unokk 2; gyerekek s hzastrsaik 4; a tzok, hzastrsa s

    az testvre 3; szleik s a tzok szleinek testvrei 6; a tzok nagyszlei 4; az szleik 8.sszesen 27.

    d) Az unokk miatt hromfle grf lehetsges: vagy mindkt unoka a tzok finak gyereke, vagymindkett a tzok lnynak gyereke, vagy egyik unoka a fi, egyik pedig a lny. Az az brahinyzik, amikor mindkt unoka a lnygyerek.

    e) Az brn jl lthat, nem fagrfot kaptunk. Amennyiben az unokk testvrek, t krt s egyizollt pontot tartalmaz a grf; ha pedig unokatestvrek, akkor ngy krt s nulla izollt pontottartalmaz a csaldfa grfja.

    f ) A tzokok nem hzasodnak csaldon bell, ezrt minden madrnak kt jabb szlje van. Mivelaz kszlkig ngy generci van, sszesen 24 = 16 kszlje van a tzoknak.

    g) Az sszes rokon ismtls nlkli permutciinak szma: 27!h) Az egyes sorokban ismtls nlkli permutcikat kell szmtanunk. Az egyes sorok nem fggnek

    a tbbitl, gy szorzatukat kell tekintennk: 2! 4! 3! 6! 4! 8!i) A 27 tzokot kell hrom csapdban elhelyeznnk, maximum tzesvel. gy a lehetsges esetek:

    10 + 10 + 7 (3), 10 + 9 + 8 (6), 9 + 9 + 9 (1).Zrjelben a darabszmok lehetsges permutciinak szmt adtuk meg. Mivel az egyesvaricik minden esetben megegyeznek, gy az sszes esetek szma egy elg nagy rtk:

    Vegyes feladatok megoldsok

    w x3138 1, 1, 0, 1, 1, 2, 3, 5, 8, 13,

    w x3139 7! = 5040.

    w x3140

    w x314112!

    != =

    ( ).

    12 312 11 10 1320

    6!

    1! 2! 3!=

    60.

    3 2710

    1710

    77

    6 2710

    179

    88

    1 279

    + +

    189

    99

    .

    nagynninagybcsi apja

    felesg

    sgor

    anyja

    aps anys

    ddszlk

    nagyszlk

    tzok

    menye fialnya veje

    unokk

    nagynninagybcsi apja

    felesg

    sgor

    anyja

    aps anys

    ddszlk

    nagyszlk

    tzok

    menye fialnya

    unoka unoka

    veje

    matematika_11_fgy_mo.qxd 2010.11.16. 9:07 Page 30

  • KOMBINATORIKA , GRFOK

    31

    w x3142 710 = 282 475 249.

    w x3143 a)

    b)

    w x3144 a) b)

    w x3145 a) b) c) d)

    w x3146 A megadott konvex sokszg tlinak szmra felrhat:

    Az egyenlet rendezse utn kapjuk:n2 3n 340 = 0,

    amibl n1 = 20 s n2 = 17. A sokszgnek teht 20 cscsa van.

    w x3147

    w x3148 a) 16x4 32x3y + 24x2y2 8xy3 + y4;

    b) a9 + 18a8b + 144a7b2 + 672a6b3 + 2016a5b4 + 4032a4b5 + 5376a3b6 + 4608a2b7 ++ 2304ab8 + 512b9;

    c)

    d)

    w x3149 a) 28 = 256; b) 216 = 65536; c) egyenlk; d) egyenlk.

    w x3150 Elszr A felhvja B-t, aki megadja D telefonszmt.Msodszor A felhvja D-t, aki megadja C szmt. Harmadszor A felhvja C-t, aki megadja E szmt. Negyedik lpsben A felhvhatja E-t: ngy hvs szksges.

    w x3151 a) Igen, a teljes grf.b) Nem.c) Kpzeljnk el egy szablyos hatszget, kssk ssze llel a szomszdos s a szemkzti pon-

    tokat.d) Nem.e) Tegyk ugyanazt, mint c)-ben.

    w x3152 sszesen 5 5 2 = 50 meghajlst fog ltni a titkrn. Az eddig ltott meghajlsok szma:2 + 3 + 1 + 4 + 5 + 3 = 18,

    azaz 50 18 = 32 mg htra van.

    A

    D

    C

    B

    E

    12

    12

    72

    14 70 224 448 5128

    83

    72

    6 2 5 3 4 4 3 5 2 6 7x x y x y x y x y x y x y xy+ + + + + + + + 2256 8y .

    3363 2378 2+ ;

    905

    50

    855

    51

    854

    1023826

    .+ =

    n nn n

    n2

    12

    170

    ( ) .= =

    42

    144

    6006

    = .

    44

    142

    91

    = ;

    40

    146

    3003

    = ;

    186

    18564 = ;

    103

    107

    120

    = = .

    107

    120 = ;

    8

    3 5

    8 7 6

    3 2 18 7 56

    11

    2 9

    11 10

    2 155

    !

    !

    !

    !.

    != = = >

    != =

    25

    3 22

    25 24 23

    3 2 125 4 23 2300

    !

    !;

    != = =

    matematika_11_fgy_mo.qxd 2010.11.16. 9:07 Page 31

  • MEGOLDSOK 11 . VFOLYAM

    32

    w x3153 a) A grf sztesik kt klnll rszre: a pross a pratlan sorszm pontokra.

    b) Jelest.

    w x3154 Nem, a grf sztesik egy BGDE s egy AHCF klnll rszre.

    w x3155 a) Sta: AFAFG. b) Vonal: ACDAEG. c) t: ABG.

    w x3156 a) A nyitott Euler-vonalhoz egy lt, a zrt Euler-vonalhoz kett kzs vgpont nlkli lt kellberajzolni az A, B, D, F pontok kztt.

    b) Az AB lt kell trlni.

    w x3157 Az bra szimmetrikus a bal fels sarokbl a jobb als sarokba hzott tlra.

    w x3158 a) ltalban fagrfot kapunk.b) Csak akkor nem fa a kapott grf, ha tartalmaz krt. Ez pedig akkor van, ha vagy a szlk, vagy

    valamelyik nagyszlk testvrek vagy fltestvrek.

    w x3159 Mivel hat helysg van, a felttelnek megfelelen fagrfot kell ltrehoznunk: pontosan t tnak(lnek) kell lenni. Most hrom van, kettt szksges pteni. Pldul lehet Andalgibl Flntbas onnan Butulba, vagy Celebbl Flntba s onnan Egygybe.

    w x3160 Kt mrs elegend: elszr vegynk el egyet, s szedjk szthrom-hrom rmre a maradkot. Ezeket mrjk meg, ha a ktrsz egyenl tmeg, ksz vagyunk, az elvett rme a hamis.Ha az egyik rsz nehezebb, ismteljk meg azon hrom rmrea fentieket. Mindez fagrffal brzolva a rajzon lthat.

    +

    + 0, a; d) a, b R, a, b 0,

    e) a > 0, f ) a, b > 0,

    g) a, b > 0,

    h) a, b > 0,

    w x3163 a) 69 93 = 29 315 > 8 187 = 210 314; b)

    c)

    d)

    e)

    f )

    w x3164 a)

    b)

    c)

    d)2000 25

    8002 5 5

    2 52

    20 11

    1915

    80 60 22

    95 3815

    ( ) ;

    = =

    24 3618 256 72

    2 3 2 32 3 2 2 3

    4 3

    4 1 2

    12 4 6 6

    4 8 8 6

    =

    ;4

    3=

    6 109 200 32

    2 3 2 53 2 5

    6 2

    3 1 1

    100 6 6 2 2

    6 3 2

    =

    2

    15

    100

    ;=

    3 10 21 256 15 35

    3 2 5 3 7 52

    5 6 5 2

    7 5 5

    5 6 6 5 5 4

    7

    =

    3 3 5 5 718

    7 5 5 5 5 ;=

    45 75 5 3 3 5 3 5

    3 51

    153

    7 21 7 14 21 42 35 49

    17 24 1

    = = =

    = > 77 25 17 245 3 515

    .=

    20 16 2 5 2 525

    2 55 8 42 5 14 1427

    941 5

    ; = < =

    100 10 10 10 1043 83 4015 95 2715 ;= = < =

    60 2 3 5 2 3 5 220106 4020 2010 20106 10201 1005 1005

    26 402= = = ( ) 00 2010 20106 3 5 ;

    7 7 49 7 7 7533 17 3 18 17 33= < = ;

    b

    a

    b

    a

    b

    a

    b

    a

    b

    a

    159

    17760

    2

    2

    39

    5760

    2

    2

    13

    1920= = .

    a b b a b17 3624 17 1224 = ;

    b

    a

    b

    a

    b

    a

    29

    2712

    2

    2

    5

    312= ;a a a2912 2 512= ;

    b

    ab

    b

    a

    73 2 3= ;

    1 1y

    y= ;13

    3

    xx= ;

    matematika_11_fgy_mo.qxd 2010.11.16. 9:07 Page 33

  • MEGOLDSOK 11 . VFOLYAM

    e)

    f )

    w x3165 a)

    b)

    c)

    d)

    Hatvnyfggvnyek s gykfggvnyek megoldsok

    w x3166 a) b) c)

    x R; x R; x R;d) e) f )

    x R; x R; x R;

    x1

    1

    1510

    5

    y

    3( ) +4 3f x x

    x1 5

    1

    5

    1

    5

    5

    y

    3( ) 1+1f x x

    x1 5

    1

    15

    5

    y

    31

    ( ) ( +2) 53

    f x x10

    x1 5

    1

    5

    10

    15

    y

    3( ) ( +3) +4f x x

    x1 5

    1

    1

    5

    y

    3( ) ( +4) 3f x x

    5

    x1 5

    1

    5

    15

    5

    y

    3( ) ( 1) +1f x x

    6 3 10 6 3 10 3 3 9 3 3 1 3 3 9 3 3 1

    3 1

    3 3 3 3

    ( ) + = + + + + =

    = +

    333 33 3 1 2 .( ) =

    8 3 49 3 7 15 6 49 12 7 7 1 7 2 13 33 3 33 333 3 33 + + + + = + + = ( ) ( ) ;;

    28 16 3 28 16 3 4 2 3 4 2 3

    4 2 3 4 2 3 3 1

    4 4 24 24 + = + =

    = + = +

    ( ) ( )

    (( ) ( )2 23 1 2 ;=

    7 4 3 7 4 3 2 3 2 3 42 2

    + + = + + = ( ) ( ) ;

    100 20 5

    1000 2

    100 20 51000 2

    53 34 712

    56 73

    20 9 7

    10 2812

    = == =2 5 2 5 5

    2 5 215

    40 40 18 9 7

    30 30 2812

    .

    45 15 273 5

    3 5 31

    15125 75 45

    24 12

    7 7 125

    : ;( )

    = =

    34

    matematika_11_fgy_mo.qxd 2010.11.16. 9:07 Page 34

  • HATVNY, GYK , LOGARITMUS

    35

    g) h) i)

    x R; x R; x R;j) k) l)

    x R; x R; x R;m) n) o) p)

    x 1, x R; x 3, x R; x 1, x R; x 2, x R.

    w x3167 a) rtkkszlete: [6; 10].Menete: a fggvny nvekszik.Szlsrtkei: minimumnak helye: x = 2, rtke: y = 6;

    maximumnak helye: x = 2, rtke: y = 10.Zrushelye: x = .23

    x

    y

    5

    5

    10

    22

    f x( )

    x1

    51

    1

    y

    4( ) 2 2+2f x x

    x1 5

    1

    1

    y

    4( ) +1+4f x x

    x1

    1

    1

    y

    4( ) +3 3f x x

    5

    x1

    51

    1

    y

    4( ) 1 2f x x

    x1 5

    1

    5

    15

    y

    4( ) 3( +1) +4f x x

    5

    x1 5

    1

    5

    15

    y

    4( ) ( 3) +6f x x

    5x1 5

    1

    5

    10

    15

    y

    4( ) ( +4) +1f x x

    x1 5

    1

    5

    15

    5

    y

    4( ) ( 2) 3f x x

    x1 5

    1

    1

    5

    y

    31

    ( ) +2 53

    f x x

    5

    x1

    1

    1510

    y

    3( ) +3+4f x x

    5

    matematika_11_fgy_mo.qxd 2010.11.16. 9:07 Page 35

  • MEGOLDSOK 11 . VFOLYAM

    b) rtkkszlete: [6; 10].Menete: [2; 0]-ban cskken,

    [0; 2]-ban nvekszik.Szlsrtkei: minimumnak helye: x = 0, rtke: y = 6;

    maximumnak helye: x = 2 s x = 2, rtke: y = 10.Zrushelyek:

    c) rtkkszlete: [2; 2].Menete: a fggvny nvekszik.Szlsrtkei: minimumnak helye: x = 6,

    rtke: y = 2;maximumnak helye: x = 10,

    rtke: y = 2.Zrushelye: x = 2.

    d) rtkkszlete: [0; 2].Menete: a fggvny nvekszik.Szlsrtkei: minimumnak helye: x = 2,

    rtke: y = 0;maximumnak helye: x = 14,

    rtke: y = 2.Zrushelye: x = 2.

    e) rtkkszlete: [1; 4].Menete: a fggvny nvekszik.Szlsrtkei: minimumnak helye: x = 5,

    rtke: y = 1;maximumnak helye: x = 4,

    rtke: y = 4.Zrushelye: nincs.

    f ) rtkkszlete: [1; 3].Menete: a fggvny cskken.Szlsrtkei: minimumnak helye: x = 20,

    rtke: y = 1;maximumnak helye: x = 4,

    rtke: y = 3.Zrushelye: nincs.

    y

    5

    1

    5

    1

    k x( )

    x10 15

    3

    4

    y

    5

    1

    1

    j x( )

    x5

    4

    4

    y

    5 10

    2

    2

    i x( )

    x2 14

    y

    55 10

    2

    2

    h x( )

    x6

    x x= s =6 64 4 .

    x

    y

    5

    5

    10

    22

    g x( )

    36

    matematika_11_fgy_mo.qxd 2010.11.16. 9:07 Page 36

  • HATVNY, GYK , LOGARITMUS

    37

    g) rtkkszlete: [1; 2].Menete: a fggvny cskken.Szlsrtkei: minimumnak helye: x = 2,

    rtke: y = 1;maximumnak helye: x = 7,

    rtke: y = 2.Zrushelye: x = 1.

    h) rtkkszlete: [2; 0].Menete: a fggvny cskken.Szlsrtkei: minimumnak helye: x = 3,

    rtke: y = 2;maximumnak helye: x = 13,

    rtke: y = 0.Zrushelye: x = 13.

    w x3168 a) A fggvny talaktva:

    rtkkszlete: [1; 7].Menete: [0; 2]-ban cskken,

    [2; 4]-ban nvekszik.Szlsrtkei: minimumnak helye: x = 2,

    rtke: y = 1;maximumnak helye: x = 0 s x = 4,

    rtke: y = 7.Zrushelyei: x = 1 s x = 3.

    b) A fggvny talaktva:

    rtkkszlete: [0; 2].Menete: [8; 1]-ban cskken,

    [1; 1]-ban konstans,[1; 8]-ban nvekszik.

    Szlsrtkei: minimumnak helye: 1 x 1,rtke: y = 0;

    maximumnak helye: x = 8 s x = 8,rtke: y = 2.

    Zrushelye: 1 x 1.

    g x

    x x

    x

    x x

    ( )

    , ,

    , ,

    , .

    =

    ha 1.Menete: a fggvny szigoran monoton cskken.Zrushelye: x = 0.

    f ) rtkkszlete: f (x) > 3.Menete: a fggvny szigoran monoton cskken.Zrushelye: x = 1.

    x

    y

    1

    1

    5

    5

    10

    15

    f x( )

    5

    x

    y

    11 5

    5

    10

    15

    e x( )

    5

    x

    y

    1

    1

    5

    5

    10

    15

    d x( )

    x

    y

    11 5

    5

    10

    15

    c x( )

    5

    40

    matematika_11_fgy_mo.qxd 2010.11.16. 9:07 Page 40

  • HATVNY, GYK , LOGARITMUS

    41

    g) rtkkszlete: g(x) > 0.Menete: a fggvny szigoran monoton nvekszik.Zrushelye: nincs.

    h) rtkkszlete: h(x) > 0.Menete: a fggvny szigoran monoton nvekszik.Zrushelye: nincs.

    i) rtkkszlete: i(x) > 2.Menete: a fggvny szigoran monoton nvekszik.Zrushelye: x = 4.

    j) rtkkszlete: j(x) > 1.Menete: a fggvny szigoran monoton nvekszik.Zrushelye: nincs.

    x

    y

    1

    1

    5

    5

    10

    15

    j x( )

    x

    y

    1 3

    1

    5

    5

    10

    15

    i x( )

    5

    x

    y

    1 2

    1

    5

    5

    10

    15

    h x( )

    x

    y

    1 4

    1

    5

    5

    10

    15

    g x( )

    matematika_11_fgy_mo.qxd 2010.11.16. 9:07 Page 41

  • MEGOLDSOK 11 . VFOLYAM

    k) rtkkszlete: k(x) > 1.Menete: a fggvny szigoran monoton nvekszik.Zrushelye: x = 1.

    l) rtkkszlete: l(x) > 1.Menete: a fggvny szigoran monoton cskken.Zrushelye: x = 2.

    m) rtkkszlete: m(x) > 5.Menete: a fggvny szigoran monoton cskken.Zrushelye: x = 4.

    n) rtkkszlete: n(x) > 3.Menete: a fggvny szigoran monoton nvekszik.Zrushelye: x = 1.

    x

    y

    1

    1

    5

    5

    15

    5

    n x( )

    x

    y

    1

    1

    5

    5

    135

    5

    m x( )

    x

    y

    11 5

    5

    10

    15

    l x( )

    5

    x

    y

    11 5

    5

    10

    15

    k x( )

    5

    42

    matematika_11_fgy_mo.qxd 2010.11.16. 9:07 Page 42

  • HATVNY, GYK , LOGARITMUS

    43

    o) Az o(x) fggvny talakthat a kvetkezkppen:

    rtkkszlete: o(x) > 1.Menete: a fggvny szigoran monoton nvekszik.Zrushelye: nincs.

    p) A p(x) fggvny talakthat a kvetkezkppen:

    rtkkszlete: p(x) > 3.Menete: a fggvny szigoran monoton nvekszik.Zrushelye: x 4,6.(Ksbb, ha a logaritmust tanuljuk, ezt az rtket mr ki tudjukszmtani.)

    w x3176 a) a(x) = 2x 2; b) b(x) = 2x 4;

    c) c(x) = 2x 1; d) d(x) = 2x + 3;

    e) e(x) = 2x + 1 + 1; f ) f (x) = 2x 2 8;

    g) g(x) = 2x 2, g(x) = h) h(x) = 2x 2 + 1, h(x) =

    i) i(x) = 23 x 4, i(x) = j) j(x) = 2x 3 8, j(x) =

    w x3177 a) t(0) = 22 C, t(6) 33 C.b) t(15) 60,76 C, teht tllpte a 60 C-ot.c) t(8) 37,82 C. t(12) 49,59 C, ami 31,1%-os nvekeds.

    w x3178 a) A fggvny talaktva:

    f xxxx

    ( ), ,, ,

    , .=

    ha >ha =ha

    w x3187 a) x = 3; b) x = 4; c) x = 1; d) x = e) x = 4; f ) x = 3;

    g) x = 2; h) x = 3; i) x1 = 1, x2 = 2; j) x1 = 4, x2 = 2;

    k) x = 1; l) x = m) x = 1; n) x = o) x = 2; p) x = 4. ;13

    23

    ;

    ;7

    11

    3217

    .2517

    ;

    59

    ; ;83

    ;13

    35

    ;12

    ;

    12

    ;

    12

    ,

    ;23

    14

    ;

    ;15

    ;52

    52

    12

    ;

    65

    ;

    ;52

    72

    ;15

    ;132

    ;

    72

    ;12

    ; ;14

    54

    ;

    matematika_11_fgy_mo.qxd 2010.11.16. 9:07 Page 45

  • MEGOLDSOK 11 . VFOLYAM

    w x3188 a) x1 = 7, x2 = 2; b) x1 = 5, x2 = 3; c) x1 = 3, x2 = 3;d) x1 = 4, x2 = 1; e) x1 = 5, x2 = 5; f ) x = 64;g) x = 2; h) x = 1; i) x = p + 2kp, k Z;

    j) x1 = + 2kp, x2 = + 2lp, k, l Z; k) x = 5.

    l) x = 17; m) x = 3; n) x1, 2 =

    o) x1 = 1, x2 = 4; p) x1 = 1, x2 = 3.

    w x3189 a) A hatvnyokat trva:

    ahonnan 3x = 1 s 2y = 8, a megolds: x = 0, y = 3.

    b) Az exponencilis fggvny szigor nvekedse miatt a kt egyenletbl:

    a megolds: x = y =

    c) Az exponencilis fggvny szigor nvekedse miatt a kt egyenletbl:

    a megolds: x = 2, y =

    d) Az exponencilis fggvny szigor nvekedse miatt a kt egyenletbl:

    a megoldsok: x1 = y1 = 1 s x2 = y2 =

    e) Jellje 6x = a, s 5y = b. gy:

    A (2)-es egyenletet beszorozva a b-vel, valamint az (1) egyenletbl kifejezett -thelyettestve (2)-be kapjuk:

    amibl ered: b2 2b + 1 = 0. Megoldsai: x = 0, y = 0.

    f ) A (2) egyenletben trendezs utn: Felhasznlva, hogy 1 = y0 (y > 0), valamint,hogy az exponencilis fggvny szigor monoton, kapjuk: x2 7y 1 = 0. Ebbe helyettestve(1)-bl kifejezett y = 1 x-et nyerjk az x2 + 7x 8 = 0 egyenletet, melynek gykei: x1 = 1,s x2 = 8, melyhez tartoz y rtkek: y1 = 0, s y2 = 9.Az egyenletrendszer megoldsa: (8 ; 9) szmpr.

    yx y2 7 1 1 .=

    5 813 8

    513

    13 85

    bb

    bb

    + = ,

    ab

    =13 8

    5

    ( )

    ( ).

    1 5 8 13

    25 8

    13

    a b

    a b

    + =

    + =

    .43

    ,12

    23

    ,( )

    ( )

    ,1

    23

    2 213

    x y

    x y

    =

    =

    34

    .( )

    ( ),

    1 2 3

    2 2 13

    xy

    xy xy

    =

    + = +

    13

    .12

    ,( )

    ( )

    ,1 2 2

    23

    2 3 112

    x y

    x y

    + =

    + =

    ( )

    ( )

    ,1 45 3

    72

    2 17

    2 12 358

    2 17

    x y

    x y

    =

    + =

    2;

    11

    6

    p76

    p

    46

    matematika_11_fgy_mo.qxd 2010.11.16. 9:07 Page 46

  • HATVNY, GYK , LOGARITMUS

    47

    w x3190 a) Az exponencilis fggvny sz